Pediatric Success for Exam 2

अब Quizwiz के साथ अपने होमवर्क और परीक्षाओं को एस करें!

What can a nurse do to reinforce a 5-vear-old's intellectual initiative when he asks about his upcoming surgery? 1. Answer the child's questions about his upcoming surgery in simple terms. 2. Provide the child with a book that has vivid illustrations about his surgery. 3. Tell the child he should wait and ask the doctor his questions. 4. Tell the child that she will answer his questions at a later time.

ANSWER 1The child is taking the initiative to ask questions, as all preschoolers do, and the nurse should always answer thos questions as appropriately and accurately as possible. 2. A book illustrating what will happen to the child may help him, but it will not encourage his intellectual initiative. 3. By not answering the child's questions, the nurse may actually be stifling his sense of initiative. 4. By not answering the child's questions, the nurse may actually be stifling his sense of initiative.

23. Which should the nurse do to prevent separation anxiety in a hospitalized toddler? 1. Assume the parental role when parents are not able to be at the bedside. 2. Encourage the parents to always remain at the bedside. 3. Establish a routine similar to that of the child's home. 4. Rotate nursing staff so the child becomes comfortable with a variety of nurses.

3. Establish a routine similar to that of the child's home. It is very important to try to maintain a child's home routine both when par- ents are present and when they have to leave the hospital. This will increase the child's sense of security and decrease anxiety. Providing consistent nursing care is impor- tant, not rotating staff.

59. The parent of a 9-month-old calls the ED because his child is choking on a marble. The parent asks how to help his child while awaiting emergency medical services. Which is the nurse's best response? 1, "You should administer five abdominal thrusts followed by five back blows." "You should try to retrieve the object by inserting your finger in your child's mouth." 3. "You should perform the Heimlich maneuver." "You should administer five back blows followed by five chest thrusts."

59. 4 1. Abdominal thrusts are not recommended for children younger than 1 year. 2. Inserting a finger in the child's mouth may cause the object to be pushed farther down the airway, making it more difficult to remove. 3. The Heimlich maneuver should be performed only on adults. 4. The current recommendation for infants younger than 1 year is to administer five back blows followed by five chest thrusts.

25. Which would be an early sign of respiratory distress in a 2-month-old? 1. Breathing shallowly. 2. Tachypnea. 3. Tachycardia. 4. Bradycardia.

2 25. 1. Shallow breathing is a late sign of respiratory distress. 2. Tachypnea is an early sign of distress and is often the first sign of respiratory illness in infants. 3. Tachycardia is a compensatory response by the body. When a child has respiratory distress and is not oxygenating well, the body increases the heart rate in an attempt to improve oxygenation. 4. Bradycardia is a late sign of respiratory distress.

31. A 3-year-old is hospitalized for an ASD repair. The parents have decided to go home for a few hours to spend time with her siblings. The child asks when her mommy and daddy will be back. The nurse's best response is: 1. "Your mommy and daddy will be back after your nap." 2. "Your mommy and daddy will be back at 6:00 p.m." 3. "Your mommy and daddy will be back later this evening." 4. "Your mommy and daddy will be back in 3 hours."

1. "Your mommy and daddy will be back after your nap." Preschoolers understand time in relation to events.

-

0.

16. Which laboratory result will provide the most important information regarding the respiratory status of a child with an acute asthma exacerbation? 1. CBC. 2. ABG. 3. BUN. 4. PTT.

2 16. 1. The CBC gives the health-care team information about the child's red and white blood cell counts and hemoglobin and hematocrit levels. The CBC indicates whether the child has or is developing an infection, but it reveals nothing about the 2. child's current respiratory status. The ABG gives the health-care team valuable information about the child's respiratory status: oxygenation level, carbon dioxide level, and blood pH. 3. The BUN provides information about the patient's kidney function, but it reveals nothing regarding the patient's respiratory status. 4. The PTT provides information about how long it takes the patient's blood to clot, but it reveals nothing about the patient's respiratory status.

20. A nurse instructs the parent of a child with sickle cell disease about factors that might precipitate a pain crisis in the child. Which of the following factors identified by the parent as being able to cause a pain crisis indicates a need for turther instruction? 1.Intection. 2. Overhydration. 3. Stress at school. 4. Cold environment.

2 20. 1. An infection can cause a child to go into crisis. 2. Overhydration does not cause a crisis. 3. Emotional stress can cause a child to go into crisis. 4. A cold environment causes vasoconstric-sion, which could lead to crisis.

5. A nurse is caring for a child with von Willebrand disease. The nurse is aware that which of the following is a clinical manitestation of von Willebrand disease? Select all that apply. 1. The child bruises easily. 2. Excessive menstruation. 3. The child has frequent nosebleeds. 4. Elevated creatinine levels. 5. Elevated blood pressure.

1, 2, 3. 1. Von Willebrand disease is a hereditary bleeding disorder characterized by deficiency of or defect in a protein. The disorder causes adherence of platelets to damaged endothelium and a mild deficiency of factor VIII. One of the manifestations of this disease is bleeding of the mucous membranes. 2. Excessive menstruation may be a manifestation of this disease. 3. Frequent nosebleeds are a common manifestation of this disease. 4. There is no increase in creatinine in this disease. 5. There is no increase in blood pressure.

3. Which of the following factors need(s) to be included in a teaching plan for a child with sickle cell disease? Select all that apply. 1. The child needs to be taken to a physician when sick. 2. The parent should make sure the child sleeps in an air-conditioned room. 3. Emotional stress should be avoided. 4. It is important to keep the child well hydrated. 5. It is important to make sure the child gets adequate nutrition.

1, 3, 4, 5 1. Seek medical attention for illness to pre vent the child from going into a crisis. 2. A cold environment causes vasoconstriction, which needs to be prevented to get good tissue perfusion. 3. Stress can cause a depressed immune system, making the child more susceptible to infection and crisis. Parents and children are advised to avoid stress. 4. The child needs good hydration andnutrition to maintain good health. 5. The child needs rood hydration and nutrition to maintain good health.

34. The parent of a child with frequent ear infections asks the nurse if there is anything that can be done to help avoid future ear infections. Which is the nurse's best response? 1. "Your child should be put on a daily dose of montelukast (Singulair)." 2. "Your child should be kept away from tobacco smoke." 3. "Your child should be kept away from other children with otitis media." 4. "Your child should always wear a hat when outside in the cold.?"

2 34. 1. Montelukast is an allergy medication, but it has not been proved to help reduce the number of ear infections in children. 2. Tobacco smoke has been proved to increase the incidence of ear infections. The tobacco smoke damages mucocil-iary function, prolonging the inflammatory process and impeding drainage through the eustachian tube. 3. Otitis media is not transmitted from one child to another. Otitis is often preceded by a URI, so children who are around other children with URIs may contract one, increasing their chances of developing an ear infection. 4. Wearing a hat outside will have no impact on whether a child contracts an ear infection.

26. Which stressor is common in hospitalized toddlers? Select all that apply. 1. Social isolation. 2. Interrupted routine. 3. Sleep disturbances. 4. Self-concept disturbances. 5. Fear of being hurt.

2. Interrupted routine. 3. Sleep disturbances. 5. Fear of being hurt.

22. Which breathing exercises should the nurse have an 3-year-old child with asthma do to increase her expiratory phase? 1. Use an incentive spirometer. 2. Breathe into a paper bag. 3. Blow a pinwheel. 4. Take several deep breaths.

22. 3 1. A child of 3 years old is too young to comply with incentive spirometry, and this activity will not increase the expiratory phase anyway. 2. Breathing into a paper bag results in a prolonged inspiratory and expiratory phase. 3. Blowing a pinwheel is an excellent means of increasing a child's expiratory phase. Play is an effective means of engaging a child in therapeutic activi-ties. Blowing bubbles is another method to increase the child's expiratory phase. 4. Taking deep breaths results in a prolonged inspiratory phase.

62 A preschooler presents to the emergency department with the following signs/symp-toms: bilateral leg purpura, eyelid edema, Edema and arthralgia of left knee, and mild bloody diarrhea. The most likely diagnosis is? 1. Inflammatory bowel disease 2. Rheumatic tever. 3. HSP. 4. ALL

3 62. 1. Inflammatory bowel disease presents with anorexia, growth retardation, joint pain, diarrhea (may be bloody), and weight loss. 2. Rheumatic fever is caused by a group A beta hemolytic streptococcus infection. It can present with cardiac murmur, fever, rash, sore throat, and joint pain. 3. HSP presents with leg or arm purpura, nausea, vomiting, bloody diarrhea, and urinary symptoms. 4. ALL presents with some of the following symptoms: fever, fatigue, joint pain, weight loss, frequent infections, and pallor.

11. A child's parent asks the nurse what treatment the child will need for the diagnosis of strep throat. Which is the nurse's best response? 1. "Your child will be sent home on bedrest and should recover in a few days without any intervention." 2. "Your child will need to have the tonsils removed to prevent future strep HIT infections." 3. "Your child will need oral penicillin for 10 days and should feel better in a few days." 4. "Your child will need to be admitted to the hospital for 5 days of IV antibiotics."

3 11. 1. The child may need bedrest. However, the child does need antibiotics to treat the strep infection. 2. The child does not need the tonsils removed; the child has strep throat. Surgical removal of the tonsils is done only following recurrent bouts of group A strep infections. 3. The child will need a 10-day course of penicillin/amoxicillin to treat the strep infection. It is essential that the nurse always tell the family that, although the child will feel better in a few days, the entire course of antibiotics must be completed. 4. Strep throat can be treated at home with oral penicillin/amoxicillin and does not require IV antibiotics and hospitalization.

The nurse is taking care of a child with sickle cell disease. The nurse is aware which of the following problems are associated with sickle cell disease. Select all that of the following problems is (are): 1. Polycythemia. 2. Hemarthrosis. 3. Aplastic crisis 4. Thrombocytopenia. 5. Vaso-occlusive criSIS.

3, 5 1. Polycythemia is seen in children with chronic hypoxia, such as evanotic heart disease. 2. Hemarthrosis, bleeding into a joint, is com-monly seen in children with hemophilia. 3. Aplastic crisis, temporary cessation of red blood cell production. is associatedwith sickle cell disease. 4 Thromboctopenia is associated with ITP, high altitude, medication side effects, and pregnancy. 5 Vaso-occlusive crisis is the most comMon problem in children with sickle cell disease.

4. A nurse is caring for a 5-year-old with sickle cell vaso-occlusive crisis. Which of the following orders should the nurse question? Select all that apply. 1. Position the child for comfort. 2 Apply hot packs to painful areas. 3 Give meperidine Demerol) 25 mg intravenously every 4 hours as needed for pain. 4 Restrict oral fluids. 5. Apply oxygen per nasal cannula to keep oxygen saturations above 94%.

3,4,5. 1. Medical treatment of sickle cell vaso-occlusive crises is directed toward preventing hypoxia. Tissue hypoxia is very painful, so placing the child in a position of comfort is important. 2. Hot packs help relieve pain because they cause vasodilation, which allows increased blood flow and decreased hypoxia. 3. Tissue hypoxia is very painful. Narcotics such as morphine are usually given for pain when the child is in a crisis. Meperidine (Demerol) should be avoided because of the risk of Demerol-induced seizures. 4. The child should receive hydration because when the child is in crisis, the abnormal S-shaped red blood cells clump, causing tissue hypoxia and pain. 5. Oxygen is of little value unless the tissue is hypoxic. The objective of treatment is to minimize hypoxia.

30. The best method to explain a procedure to a hospitalized preschool-age child is to: 1. Show the child a pamphlet with pictures showing the procedure. 2. Have the 5-year-old next door tell the 4-year-old about the experience. 3. Demonstrate the procedure on a doll. 4. Show the child a video of the procedure

3. Demonstrate the procedure on a doll. A 4-year-old child understands in very concrete and simple terms. Therefore, medical play is an excellent method for helping to understand the procedure.

33. Which reaction would a nurse expect when giving preschooler immunizations? 1. Silent and still. 2. Cries and tells the nurse it hurts. 3. Stalling. 4. Reamins still while telling the nurse she is hurting him.

33. 1. Teens are more likely to be stoic and remain still and silent during injections. ANS The common response of a 5-year-old is to cry and protest during an immunization. 3. School-age children are most likely to try to stall the nurse. Teens usually remain still, and they may calmly tell the nurse that they are feeling pain during the injection.

35. Which child would benefit most from having ear tubes placed? Select all that apply. 1. A 9-month-old who has had one ear infection. 2. A 13-month-old with recurrent ear infections. 3. A 2-year-old who has had five previous ear infections. 4. A 3-year-old whose sibling has had four ear infections. 5. A 7-year-old who has had two ear infections this year.

35. 2,3. 1. Surgical intervention is not a first line of treatment. Surgery is usually reserved for children who have suffered from recurrent ear infections. 2. A child with recurrent ear infections is a candidate for ear tubes. 3. A 2-year-old who has had multiple ear infections is a perfect candidate for ear tubes. The other issue is that a 2-year-old is at the height of language development, which can be adversely affected by recurrent ear infections because they can interfere with hearing. 4. Surgery is not a prophylactic treatment. Just because the sibling has had several ear infections does not suggest that the 3-year-old will also have frequent ear infections. The 3-year-old has not had an ear infection yet. 5. A 7-year-old who has had two ear infections is not the appropriate candidate. Surgical intervention is usually reserved for children who have suffered from recurrent ear infections.

14. A school-age child is admitted to the hospital for a tonsillectomy. During the nurse's postoperative assessment, the child's parent tells the nurse that the child is in pain. Which of the following observations would be of most concern to the nurse? 1. The child's heart rate and blood pressure are elevated. 2. The child complains of having a sore throat. 3. The child is refusing to eat solid foods. 4. The child is swallowing excessively.

4 14. 1. The patient is complaining of pain, an elevated heart rate and blood pressure are not unusual. The nurse should address the pain by giving any PRN pain medications ordered or calling the health-care provider for an order. 2. Most children will complain of pain after a tonsillectomy. This is expected. 3. Oral intake is usually limited to Popsicles, ice chips, and cold liquids following a ton-sillectomy. The child is in pain and should not be expected to be eating solid foods 8 hours after surgery. 4. Excessive swallowing is a sign that the child is swallowing blood. This should be considered a medical emergency, and the health-care provider should be contacted immediately. The child is likely bleeding and will need to return to surgery.

22. Which of the following will be abnormal in a child with the diagnosis of hemophilia? 1. Platelet count. 2. Hemoglobin level. 3. White blood cell count. 4. Partial thromboplastin time (PTT).

4 22. 1. Platelet function is normal in hemophilia. 2. There is no change in hemoglobin with a diagnosis of hemophilia. The hemoglobin will drop with bleeding. 3. The white blood cell count does not change with hemophilia. 4. The abnormal laboratory results in hemophilia are related to decreased clotting function. Partial thromboplas-tin time is prolonged.

-.

4-

40. A 4-year-old boy has been hospitalized because he fell down the stairs. His mother is crying and states, "This is all my fault." Which is the nurse's best response? 1. "Accidents happen. You shouldn't blame yourself.» 2. "Falls are one of the most common injuries in this age-group?" 3. "It may be a good idea to put a gate on the stairs." 4. "Your son should be proficient at walking down the stairs by now."

40. 1. This comment will not make the mother feel any better. The mother is going to blame herself regardless of where the blame lies. The nurse would do better to just listen than to make this sort of comment. ABSWER 2 Falls are one of the most common inju-ries, and it may make the parent feel better to know that this is common. 3. It may be a good idea to put up a gate, but in this situation the nurse's comment may be interpreted as judgmental. 4. Children walk, climb stairs, and run without paying attention to what might be in their way and can fall easily.

42. Which statements would indicate to the nurse that a school-age child is not developmentally on track for age? Select all that apply. 1. The child is able to follow a four- to five-step command. 2. The child started wetting the bed on admission to the hospital. 3. The child has an imaginary friend named Kelly. 4. The child enjoys playing board games with her sister. 5. The child is not able to follow rules.

42. 3, 5. 1. School-age children should be able to follow a four- to five-step command, so this does not indicate that the child has a developmental delay. 2. The child was potty-trained before entering the hospital, and bedwetting is a common form of regression seen in hospitalized children. The child will likely return to her normal toileting habits when she returns home. 3. Most school-age children do not have imaginary friends. This is much more common for children of 3 and 4 years of age. 4. Most school-age children do enjoy playing board games. 5. Most school-age children like the structure provided by rules and understand the consequences of not obeying them.

46. The mother of 10-year-old fraternal twins tells the nurse at their well-child checkup that she is concerned because her daughter has gained more weight and height than her twin brother. The mother is concerned that there is something wrong with her son. The nurse's best response is: 1. "I understand your concern. I will talk with the physician, and we can draw blood for laboratory work." 2. "Let me ask you whether your son has been ill lately." 3. "It is normal for girls to grow a little taller and gain more weight than boys at this age." 4. "It is normal for you to be concerned, but I am sure your son will catch up with your daughter eventually."

46. 1. This is not an appropriate response. The nurse should be aware that it is normal for girls to grow taller and gain more weight than boys near the end of middle childhood. 2. This is not an appropriate response. The nurse should be aware that it is normal for girls to grow taller and gain more weight than boys near the end of middle childhood. ANS 3. This is the appropriate response. The nurse understands that it is normal for girls to grow taller and gain more weight than boys near the end of middle childhood. This is not the best response. The boy will likely surpass his sister when he reaches adolescence.

47. A 9-year-old girl builds a clubhouse in her backyard. She hangs a sign outside her clubhouse that has "No bovs allowed" printed on it. The child's parents are concerned that she is excluding their neighbor's son, and they are upset. What should the school nurse tell the child's parents? 1. Her behavior is cause for concern and should be addressed. 2. This behavior is common among school-age children. 3. Her feelings about boys will subside within the next year. 4. They should have their daughter speak with the school counselor.

47. 1. The child's behavior is normal. Girls of 9 and 10 generally prefer to have friends who are of the same gender. ANS 2 This is common behavior. Girls of 9 and 10 generally prefer to have friends who are of the same gender. 3. Girls of 9 and 10 generally prefer to have friends who are of the same gender. The child will likely have the same feelings next year. 4. There is no need for the child to see the school counselor. Girls of 9 and 10 generally prefer to have friends who are of the same gender.

What information shoals retires ire indule in a deusion on natition with a fourth-grade class? 2. The number of calorics that a fourth-grade child should consume in a day. 2. A list of high-calorie foods that all fourth-graders should avoid. 3. How to read cold lit Egods that al fourth graders sho toods are good for there. 4. A list of nutritious foods with basic serenkow which too about how they affect the body organs and systems.

48. 1. School-age children do not engage in calorie counting. This is an adult activity. 2. Children may not want to hear this infor-mation, as most of them enjoy consuming high-calorie foods that taste good. 3. School-age children do not engage in calorie counting. This is an adult activity. ANS 4. Reviewing nutritious choices keeps the lesson on a positive note, and school-age children are very interested in how food affects their bodies. They are capable of understanding basic medical terminology.

6. A child with hemophilia A fell and injured a knee while playing outside. The knee is swollen and painful. Which of the following measures should be taken to stop the bleed-ing? Select all that apply. 1. The extremity should be immobilized. 2. The extremity should be elevated. 3. Warm moist compresses should be applied to decrease pain. 4. Passive range-of-motion exercises should be administered to the extremity. 5. Factor VIM should be administered.

6. 1, 2, 5. 1. Measures are needed to induce vasoconstriction and stop the bleeding, including immobilization of the extremity. 2. Measures are needed to induce vasoconstriction and stop the bleeding. Treatment should include elevating the extremity. 3. Measures are needed to induce vasoconstriction and stop the bleeding. Treatment should include an application of cold com-pression. Warmth will slow the vasoconstriction and clotting process. 4. Measures are needed to induce vasoconstriction and stop the bleeding. Passive ROM is not started until the bleeding has stopped. 5. Hemophilia A is a deficiency in factor VIII, which causes delay in clotting when there is a bleed. Giving a dose of factor VI concentrate will assist in the clotting process.

61. A toddler has been diagnosed with iron-deficiency anemia and is to start iron supple-mentation. Which of the following is appropriate parental teaching? Select all that apply. 1. "Be sure to give the iron with juice."« 2 Take with milk 3. "You can give the iron with a straw." 4. "Be sure to bring him back in one month to recheck his hemoglobin. 5 "Iron causes loose stools."

61. 1,3, 4. 1. Iron-deficiency anemia is common in toddlers usually due to drinking too much milk and not enough iron-rich foods. Iron is absorbed best when taken with juice. 2. Iron is not absorbed well when taken with milk, so suggest juice instead. 3. The child should come back in a month for a blood test to see if the iron supplementation is raising the iron level. 4. Iron can cause teeth to be stained, so if the toddler will drink from a straw, that is a good way to administer it. 5. Iron supplementation most often causes constipation not loose stools.

51. A 5-year-old boy has always been one of the shortest children in class. His mother tells the school nurse that her husband is 6' tall and she is 517". What should the nurse tell the child's mother? 1. He is expected to grow about 2 inches every year from ages 6 to 9 years. 2. He is expected to grow about 3 inches every year from ages 6 to 9 years. 3. He should be seen by an endocrinologist for growth-hormone injections. 4. His growth should be re-evaluated when he is 7 years old.

ANS During the school-age years, a child grows approximately 2 inches per year. 2. During the school-age years, a child grows approximately 2 inches per year. 3. This is not the appropriate time to have the child evaluated. His mother needs to reserve her concerns until he is older. He will likely begin to catch up with his peers within the next year. 4. The child should continue to see his pediatrician for annual visits, but there is no need for a special visit to re-evaluate his growth at this time.

19. Which statement by the mother of an 18-month-old would lead the nurse to believe that the child should be referred for further evaluation for developmental delay? 1. "My child is able to stand but is not yet taking steps independently." 2. "My child has a vocabulary of approximately 15 words." 3. "My child is still sucking his thumb." 4. "My child seems to be quite wary of strangers."

1. "My child is able to stand but is not yet taking steps independently." The child should be walking indepen- dently by 15 to 18 months. Because this toddler is 18 months and not walking, a referral should be made for a develop- mental consult. The vocabulary of an 18-month-old should be 10 words or more. Thumb-sucking is still common for 18-month-olds and may actually be at its peak at that age. It is very common for a child of 18 months to exhibit stranger anxiety.

37. Which protrusion into the groin of a female most likely causes inguinal hernias? 1. Bowel. 2. Fallopian tube. 3. Large thrombus formation. 4. Muscle tissue.

2 37. 1. Bowel is the most common tissue to protrude into the groin in males. 2. Fallopian tube or an ovary is the most common tissue to protrude into the groin in females. 3. Large thrombus formation does not commonly protrude into the groin. 4. Muscle tissue does not commonly protrude into the groin.

61. The parents of a 3-year-old are concerned that the child is having "more accidents" during the day. Which questions would be appropriate for the nurse to ask to obtain more information? Select all that apply. 1. "Has there been a stressful event in the child's life, such as the birth of a sibling?" 2. "Has anyone else in the family had problems with accidents?" 3. "Does your child seem to be drinking more than usual?" 4. "Is your child fussier that usual, and does your child seem to be in pain when urinating?" 5. "Is your child having difficulties at preschool?"

61. 1, 2, 3, 4. 1. Stressors such as the birth of a sibling can lead to incontinence in a child who previously had bladder control. 2. À pattern of enuresis can often be seen in families. 3. Increased thirst and incontinence can be associated with diabetes. 4. Fussiness and incontinence can be associated with UTIs. 5. Preschool-age children may have difficulties at school, but the other information would be of more help to the nurse in gathering more information on enuresis.

26. Which is a care priority for a newborn diagnosed with bladder exstrophy and a malformed pelvis? 1. Change the diaper frequently and assess for skin breakdown. 2. Keep the exposed bladder open in a warm and dry environment to avoid any heat loss. 3. Offer formula for infant growth and fluid management. 4. Cluster all care to allow the child to sleep, grow, and gain strength for the upcoming surgical repair.

1 26. 1. Preventing infection from stool contamination and skin breakdown is the top priority of care. Protect the bladder mucosa with a film wrap to keep the mucosa moist. Do not keep it open where it can be exposed to pathogens or subject to irritation from drying. Infant formula would not provide enough fluid for this infant. An umbilical artery catheter would be inserted to provide fluids because of large insensible fluid losses from the exposed viscera. 4. Although the child should be encouraged to rest, it is important to change the diaper immediately to prevent fecal contamination and subsequent infection.

27. A teen comes into the clinic with anxiety. Over the past 2 weeks, the teen has had some muscle twitching and has a positive Chostek sign. Which explanation could the nurse provide to the parent about a Chrostek sign? 1. It is a facial muscle spasm elicited by tapping the facial nerve. 2. It is muscle pain that occurs when a muscle is touched. 3. The sign occurs because of increased intracranial pressure. 4. The sign is a result of a vitamin D overdose.

1 27. 1. Chvostek sign is a facial muscle spasm elicited by tapping on the facial nerve in the region of the parotid gland, indicates heightened neuromuscular activity, and leads the nurse to suspect hypoparathyroidism. 2. Chvostek sign is a facial muscle spasm elicited by tapping on the facial nerve in the region of the parotid gland, indicates heightened neuromuscular activity, and leads the nurse to suspect hypoparathyroidism. 3. Chostek sign is a facial muscle spasm elicited by tapping on the facial nerve in the region of the parotid gland, indicates heightened neuromuscular activity, and leads the nurse to suspect hypoparathyroidism. 4. Chvostek sign is a facial muscle spasm elicited by tapping on the facial nerve in the region of the parotid gland, indicates heightened neuromuscular activity, and leads the nurse to suspect hypoparathyroidism.

35. Which causes the symptoms in testicular torsion? 1. Twisting of the spermatic cord interrupts the blood supply. 2. Swelling of the scrotal sac leads to testicular displacement. 3. Unmanaged undescended testes cause testicular displacement. 4. Microthrombi formation in the vessels of the spermatic cord causes interruption of the blood supply.

1 35. 1. Testicular torsion is caused by an interruption of the blood supply due to twisting of the spermatic cord. 2. Swelling of the scrotal sac occurs because of testicular torsion; it is not a cause of testicular torsion. 3. Unmanaged undescended testes may be a risk factor for but are not a cause of testicular torsion. 4. Microthrombi formation in the vessels of the spermatic cord does not occur in testicular torsion.

11. The parents of a child with glomerulonephritis ask how they will know their child is improving after they go home. Which are the nurse's best responses? Select all that apply. 1. "Your child's urine output will increase, and the urine will become less tea-colored." 2. "Your child will have more energy as laboratory tests become more normal." 3. "Your child's appetite will decrease as urine output increases." 4. "Your child's laboratory values will become more normal." 5. "Your child's weight will increase as the urine becomes less tea colored

11. 1, 5. 1. When glomerulonephritis is improving, urine output increases, and the urine becomes less tea-colored. These are signs that can be monitored at home by the child's parents. 2. As glomerulonephritis improves, the child should have more energy and require less rest, but laboratory test values are not something that will be readily apparent to the family at home. 3. The child's appetite should increase as the condition improves. 4. Although the laboratory test values will normalize, this is not something that will be readily apparent to the family at home. 5. The child's weight will increase as the urine resumes a more normal color, indicating laboratory values are returning to normal and the child is better. TEST-TAKING HINT: The test taker should be led

27. By two years of age, a toddler should be able to: 1. go up and down the stairs without assistance 2. speak in two to three word sentences 3. draw a circle 4 skip on alternate feet

2

29. A toddler would need a developmental screening referral If he does not: 1. run well 2. transfer an object from one hand to another 3. build a tower of six or seven cubes 4. have a vocab of 300 words

2

13. The clinical manifestations of MCNS are caused by which of the following? 1. Chemical changes in the composition of albumin. 2. Increased permeability of the glomeruli. 3. Obstruction of the capillaries of the glomeruli. 4. Loss of the kidneys' ability to excrete waste and concentrate urine.

2 13. 1. Albumin does not undergo any chemical changes in MCNS. 2. Increased permeability of the glomeruli in MCNS allows large substances such as protein to pass through and be excreted in the urine. 3. Obstruction of the capillaries of the glomeruli due to the formation of antibody-antigen complexes occurs in glomerulonephritis. 4. Loss of the kidneys' ability to excrete waste and concentrate urine occurs in renal failure.

18. A 6-year-old is diagnosed with growth hormone deficiency. A prescription is written for a dose of 0.025 mg/kg of somatotropin subcutaneously three times weekly. The child weighs 59.4 lb. Which dose of medication should the nurse administer three times weekly? 1. 0.5 mg. 2. 0.675 mg. 3. 1 mg 4. 2 mg.

2 18. 1. Change the weight in pounds to kilograms by dividing the pounds by 2.2: 59.4 ÷ 2.2 = 27 kg. Determine the dosage of medication by multiplying 27 kg by the medication prescribed at 0.025 mg/kg. The total dosage needed is 0.675 mg. 2. Change the weight in pounds to kilograms by dividing the pounds by 2.2: 59.4 ÷ 2.2 = 27 kg. Determine the dosage of medication by multiplying 27 kg by the medication prescribed at 0.025 mg/kg. The total dosage needed is 0.675 mg. 3. Change the weight in pounds to kilograms by dividing the pounds by 2.2: 59.4 ÷ 2.2 = 27 kg. Determine the dosage of medication by multiplying 27 kg by the medication prescribed at 0.025 mg/kg. The total dosage needed is 0.675 mg. 4. Change the weight in pounds to kilograms by dividing the pounds by 2.2: 59.4 ÷ 2.2 = 27 kg. Determine the dosage of medication by multiplying 27 kg by the medication prescribed at 0.025 mg/kg. The total dosage needed is 0.675 mg.

19. Somatotropin comes in a vial of 5 mg and is mixed with a diluent of 5 mL. There is 5 mL of solution in each vial. What amount of the solution should the nurse draw up to give a 1 mg dose? 1. 0.5 mL. 2. 0.675 mL. 3. 1 mL. 4. 2 mL.

2 19. 1. Determine the amount of solution by setting up an equation of 5 mg/5 mL as 0.675 mg/X mL = 0.0675 mL. 2. Determine the amount of solution by setting up an equation of 5 mg/5 ml as 0.675 mg/X mL = 0.0675 ml. 3. Determine the amount of solution by setting up an equation of 5 mg/5 mL as 0.675 mg/X mL = 0.0675ml. 4. Determine the amount of solution by setting up an equation of 5 mg/5 mL as 0.675 mg/X mL = 0.0675 mL.

24. Which needs to be present to diagnose HUS? 1. Increased red blood cells with a low reticulocyte count, increased platelet count, and renal failure. 2. Decreased red blood cells with a high reticulocyte count, decreased platelet count, and renal failure. 3. Increased red blood cells with a high reticulocyte count, increased platelet count, and renal failure. 4. Decreased red blood cells with a low reticulocyte count, decreased platelet count, and renal failure.

2 24. 1. The triad in HUS includes decreased red blood cells (with a high reticulocyte count as the body attempts to produce more red blood cells), decreased platelet count, and renal failure. 2. The triad in HUS includes decreased red blood cells (with a high reticulocyte count as the body attempts to produce more red blood cells), decreased platelet count, and renal failure. 3. The triad in HUS includes decreased red blood cells (with a high reticulocyte count as the body attempts to produce more red blood cells), decreased platelet count, and renal failure. 4. The triad in HUS includes decreased red blood cells (with a high reticulocyte count as the body attempts to produce more red blood cells), decreased platelet count, and renal failure.

48. A child had a tonsillectomy 6 days ago and was seen in the emergency department 4 hours ago because of postoperative hemorrhage. The parent noted that her child was "swallowing a lot and finally began vomiting large amounts of blood." The child's vital signs are as follows: T 99.5°F (37.5°C), HR 124, BP 84/48, and RR 26. The nurse knows that this child is at risk for which type of renal problem? 1. CKD due to advanced disease process. 2. Prerenal failure due to dehydration. 3. Primary kidney damage due to a lack of urine flowing through the system. 4. Postrenal failure due to a hypotensive state.

2 48. 1. CKD occurs gradually. 2. Examples of causes of prerenal failure include dehydration and hemorrhage. 3. Primary kidney failure occurs when the kidney experiences a direct injury. Examples include HUS and glomerulonephritis. 4. Postrenal failure occurs when there is an obstruction to urinary flow. Hypotension does not cause postrenal failure.

20. Which would the nurse most likely find in the history of a child with HUS? Select all that apply. 1. Frequent UTIs and possible VUR. 2. Vomiting and diarrhea before admission. 3. Bee sting and localized edema of the site for 3 days. 4. Previously healthy with no signs of illness. 5. Anorexia and bruising.

20. 2, 5. 1. Frequent UTIs and VUR do not lead to HUS. 2. HUS is often preceded by diarrhea that may be caused by E. coli present in undercooked meat. 3. Insect stings are not associated with HUS. 4. HUS is usually preceded by diarrhea. 5. Anorexia and bruising (purpura and/ or petechiae) are common clinical manifestations.

21. The manifestations of HUS are due primarily to which event? 1. The swollen lining of the small blood vessels damages the red blood cells, which are then removed by the spleen, leading to anemia. 2. There is a disturbance of the glomerular basement membrane, allowing large proteins to pass through. 3. The red blood cell changes shape, causing it to obstruct microcirculation. 4. There is a depression in the production of all formed elements of the blood.

21. 1 1. The swollen lining of the small blood vessels damages the red blood cells, which are then removed by the spleen. 2. The increased permeability of the basement membrane occurs in MCNS. 3. The red blood cell changing shape is typical of sickle cell anemia. 4 The depression of all formed elements of the blood occurs in aplastic anemia.

24. Which hormone(s) does the anterior pituitary secrete? Select all that apply. 1. Thyroxine. 2. Luteinizing hormone. 3. Prolactin (luteotropic hormone). 4. ACTH. 5. Epinephrine. 6. Cortisol.

24. 2, 3, 4. 1. Thyroxine is secreted by the thyroid gland. 2. Luteinizing hormone, prolactin, and ACTH are secreted by the anterior pituitary. 3. Luteinizing hormone, prolactin, and ACTH are secreted by the anterior pituitary. 4. Luteinizing hormone, prolactin, and ACTH are secreted by the anterior pituitary. 5. Epinephrine is secreted by the adrenal medulla. 6. Cortisol is secreted by the adrenal cortex.

10. A nurse is working with a child who has had a bone age evaluation. Which explanation of the test should the nurse give? 1. "The bone age will give you a diagnosis of your child's short stature." "If the bone age is delayed, the child will continue to grow taller." 3. "The agray of the bones is compared with that of the age-appropriate, standardine 4. "If the bone age is not delayed, no further treatment is needed."

3 10. 1. Bone age is a method of assessing skeletal maturity and does not give a diagnosis. 2. The child with a delayed bone age may continue to grow. In many children with a delayed bone age, medication is required to help them continue to grow. 3. The bone age is a method of evaluating the epiphyseal growth centers of the bone using standardized, age-appropriate tables. 4. A bone age that is not delayed in a child of short stature is more concerning than if the growth were delayed. This means that further testing and evaluation are needed if the growth delay is to be reversed.

22. Which laboratory results besides hematuria are most consistent with HUS? 1. Massive proteinuria, elevated blood urea nitrogen and creatinine. 2. Mild proteinuria, decreased blood urea nitrogen and creatinine. 3. Mild proteinuria, increased blood urea nitrogen and creatinine. 4. Massive proteinuria, decreased blood urea nitrogen and creatinine.

3 22. 1. Protein is not lost in massive amounts in HUS. 2. BUN and creatinine are usually increased in HUS. 3. Hematuria, mild proteinuria, and increased BUN and creatinine are all present in HUS. 4. Protein is not lost in massive amounts in HUS.

34. An adolescent woke up complaining of intense pain and swelling of the scrotal area and abdominal pain. He has vomited twice. Which should the nurse suggest? 1. Encourage him to drink clear liquids until the vomiting subsides; if he gets worse, bring him to the emergency department. 2. Bring him to the health-care provider's office for evaluation. 3. Take him to the emergency department immediately. 4. Encourage him to rest; apply ice to the scrotal area and go to the emergency department if the pain does not improve.

3 34. 1. The child is having symptoms of testicular torsion, which is a surgical emergency and needs immediate attention. The child should not wait to go to the emergency department and should be told not to drink anything in anticipation of surgery. 2. Testicular torsion is a surgical emergency, and time should not be wasted at the health-care provider's office when the child needs surgery. 3. The child is having symptoms of testicular torsion, which is a surgical emergency and needs immediate attention. 4. The child should be brought to the emergency department immediately because testicular torsion is a surgical emergency. Ice and scrotal support can be used for relief of discomfort, but bringing the child to the emergency department is the priority.

42. How will a child with respiratory distress and stridor who is diagnosed with RSV be treated? 1. IV antibiotics. 2. IV steroids. 3. Nebulized racemic epinephrine. 4. Alternating doses of acetaminophen (Tylenol) and ibuprofen (Motrin).

3 42. 1. RSV is a viral illness and is not treated with antibiotics. 2. Steroids are not used to treat RSV. 3. Racemic epinephrine promotes mucosal vasoconstriction. 4. Acetaminophen and ibuprofen can be given to the child for comfort, but they do not improve the child's respiratory status.

14. The parents of a child hospitalized with MCNS ask why the last blood test revealed elevated lipids. Which is the nurse's best response? 1. "If your child had just eaten a fatty meal, the lipids may have been falsely elevated." 2. "It's not unusual to see elevated lipids in children because of the dietary habits of today." 3. "Because your child is losing so much protein, the liver is stimulated and makes more lipids." 4. "Your child's blood is very concentrated because of the edema, so the lipids are falsely elevated."

3 14. 1. In MCNS, the lipids are truly elevated. Lipoprotein production is increased because of the increased stimulation of the liver caused by hypoalbuminemia. 2. The elevated lipids are unrelated to the child's dietary habits. 3. In MCNS, the lipids are truly elevated. Lipoprotein production is increased because of the increased stimulation of the liver caused by hypoalbuminemia. 4. The lipids are not falsely elevated.

2. What key information should be explained to the family of a 3-year-old who has short stature and abnormal laboratory test results? 1. Because of the diurnal rhythm of the body, growth hormone levels are elevated following the onset of sleep. 2. Exercise can stimulate growth hormone secretion. 3. The initial screening tests need to be repeated for accuracy. 4. Growth hormone levels in children are so low that stimulation testing must be done.

4 2. 1. Growth hormone levels elevate during sleep but cannot be used to determine a definitive diagnosis. 2. Exercise increases growth hormone secretion but cannot be used for definitive diagnosis. 3. Screening tests assist in determining in which direction further studies should be done 4. The need for additional testing requires explanation. The abnormal IGF-1 and insulin-like growth factor binding protein require a definitive diagnosis when the levels are either abnormally high or abnormally low. Very young children do not secrete adequate levels of growth hormone to measure accurately and thus require challenge/stimulation testing.

27. Which medication would most likely be included in the postoperative care of a child with repair of bladder exstrophy? 1. Furosemide (Lasix). 2. Mannitol. 3. Meperidine (Demerol). 4. Oxybutynin (Ditropan).

4 27. 1. Furosemide is a loop diuretic that is not routinely used in the care of the child with a repair of bladder exstrophy. 2. Mannitol is an osmotic diuretic that is not routinely used in the care of the child with a repair of bladder exstrophy. 3. Meperidine is a narcotic that is not a first-line drug for pain management after a bladder reconstruction. 4. Oxybutynin is used to help control bladder spasms.

4. A nurse is caring for an infant who is very fussy and has a diagnosis of DI. Which parameters should the nurse monitor while the infant is on fluid restrictions? Select all that apply. 1. Oral intake. 2. Urine output. 3. Appearance of the mucous membranes. 4. Change in pulse and temperature. 5. Lethargy and pain.

4. 2, 5. 1. Although monitoring fluid intake is neces-sary, the child is on fluid restriction, and the amount of intake will be prescribed. 2. It is crucial to monitor and record urine output. The infant with DI has hypose-cretion of ADH, and fluid restriction has little effect on urine formation. This infant is at risk for dehydration and for fluid and electrolyte imbalances. 3. It is a basic part of assessing the infant with fuid restriction to monitor skin turgor and the appearance of the mucous membranes, but they are not an absolute determination of overall well-being. 4. It is a basic part of assessing the infant to check pulse and temperature (the infant can become very hyperthermic), but neither would be an absolute indicator of well-being. 5. As a result of DI, the infant may have behavior changes such as lethargy and pain.

What would the nurse advise the parent of a child with a barky cough that gets worse at night. 1. Take the child outside into the more humid night air for 15 minutes. 2. Take the child to the ED immediately. 3. Give the child an over-the-counter cough suppressant. 4. Give the child warm liquids to soothe the throat.

43. 1 1. The humid night air will help decrease subglottic edema, easing the child's respiratory effort. The coughing should diminish significantly, and the child should be able to rest comfortably. If the symptoms do not improve after taking the child outside, the parent should have the child seen by a health-care provider. 2. There is no immediate need to bring the child to the ED. The child's symptoms will likely improve on the drive to the hospital because of the child's exposure to the night air. 3. Over-the-counter cough suppressants are not recommended for children because they reduce their ability to clear secretions. 4. Warm liquids may increase subglottic edema and actually aggravate the child's symptoms. Cool liquid or a Popsicle is the best choice.

44. Which child is in the greatest need of emergency medical treatment? 1. A 3-year-old who has a barky cough, is afebrile, and has mild intercostal retractions. 2. A 6-year-old who has high fever, no spontaneous cough, and froglike croaking. 3. A 7-year-old who has abrupt onset of moderate respiratory distress, a mild fever, and a barky cough. 4. A 13-year-old who has a high fever, stridor; and purulent secretions.

44. 2 This child has signs and symptoms of acute laryngitis and is not in a significant amount of distress. 2. This child has signs and symptoms of epiglottitis and should receive immediate emergency medical treatment. The patient has no spontaneous cough and has a froglike croaking because of a significant airway obstruction. 3. This child has signs and symptoms of LTB and is not in significant respiratory distress. 4. This child has signs and symptoms of bacterial tracheitis and should be treated with antibiotics but is not the patient in the most significant amount of distress.

What does the therapeutic management of CF patients include? Select all that apply. 1. Providing a high-protein, high-calorie diet. 2. Providing a high -fat, high-carbohydrate diet. Encouraging exercise. 4. Minimizing pulmonary complications. 5. Encouraging medication compliance.

61. 1, 3, 4, 5. 1. Children with CF have difficulty absorbing nutrients because of the blockage of the pancreatic duct. Pancreatic enzymes cannot reach the duodenum to aid in digestion of food. These children often require up to 150% of the caloric intake of their peers. The nutritional recommendation for CF patients is a high-calorie and high-protein diet. 2. A high-fat, high-carbohydrate diet is not recommended for adequate nutrition. 3. Exercise is effective in helping CF patients clear secretions. 4. Minimizing pulmonary complications is essential to a better outcome for CF patients. Compliance with CPT, nebu-lizer treatments, and medications are all components of minimizing pulmonary complications. 5. Medication compliance is a necessary part of maintaining pulmonary and gastrointestinal function.

9. The parent of a child with glomerulonephritis asks the nurse why the urine is such a funny color. Which is the nurse's best response? 1. "It is not uncommon for the urine to be discolored when children are receiving steroids and blood pressure medications." "There is blood in your child's urine that causes it to be tea-colored." 3. "Your child's urine is very concentrated, so it appears to be discolored." 4. "A ketogenic diet often causes the urine to be tea-colored."

9. 1. Steroids and antihypertensives do not cause urine to change color. 2. Blood in the child's urine causes it to be tea-colored. 3. The tea color of the urine is due to hematu-ria, not concentration. 4. The child with glomerulonephritis is not on a ketogenic diet. The ketogenic diet does not make the urine change color.

22 . A 2-year-old was admitted to the hospital 2 days ago has been crying and is inconsolable much of the time. The nurse's best response to the child's parents who are concerned about this behavior is that the child is: 1. Detachment phase of separation anxiety, which is normal for children during hospitalization. 2. Despair stage of separation anxiety, which is normal for children during hospitalization. 3. Bargaining stage of separation anxiety, which is normal for children during hospitalization. 4. Protest stage of separation anxiety, which is normal for children during hospitalization.

Answer: 4. Protest stage of separation anxiety, which is normal for children during hospitalization. Rationale: During the protest stage of separation anxiety, children are often inconsolable and cry more than they do when they are at home. these children also frequently ask to go home.

Which should the nurse recommend to the parents of a 9-year-old hospitalized following a bicycle injury? To prevent future injury, their child should: 1. Wear safety equipment while riding bicycles. 2. Read educational material on bicycle safety. 3. Watch a video on bicycle safety. 4. Ride his bike in the presence of adults.

ANS Safety equipment is essential for bicy-cling, skateboarding, and participating in contact sports. Most injuries occur during the school-age years, when children are more active and participate in contact sports. 2. Educational material is a good way to reinforce the use of safety equipment, but the parents must insist that the child use his safety equipment. 3. Video material is a good way to reinforce the use of safety equipment, but the parents must insist that the child use his safety equipment. 4. The child's parents may not always be present when he rides his bike, so the use of safety equipment is the primary concern.

A 4-year-old hospitalized with FTT has orders for daily weights, strict intake and out-put, and calorie counts. Which action by the nurse would be a concern? 1. The nurse weighs the child every morning after breakfast. 2. The nurse weighs the child with no clothing except for undergarments. 3. The nurse sits with the child while the child eats her meals. 4. The nurse weighs the child using the same scale every morning.

ANSWER The child should be weighed every day on the same scale before eating. Her weight will not be an accurate reflection if she is fed prior to being weighed. 2. The child should be weighed onlv in undergarments. The weight of clothing must not be included. 3. The nurse should remain in the room while the child eats in order to accurately record a calorie count. 4 The child should be weighed on the same scale every time. All scales are not equally accurate, so it is important to use the same scale in order to obtain an accurate trend.

1. The bladder capacity of a 3-year-old is approximately how much? 1. 1.5 fl. oz. 2. 3 fl. oz. 3. 4 fl. oz. 4. 5 1. oz.

4 1. 1. The capacity of the bladder in fluid ounces can be estimated by adding 2 to the child's age in years. 2. The capacity of the bladder in fluid ounces can be estimated by adding 2 to the child's age in years. 3. The capacity of the bladder in fluid ounces can be estimated by adding 2 to the child's age in years. 4. The capacity of the bladder in fluid ounces can be estimated by adding 2 to the child's age in years.

12. A child is brought to the ED with what is presumed to be acute adrenocortical insufft-ciency. Which of the following should the nurse do first? 1. Insert an IV line to administer fluids and cortisol. 2. Prepare for admission to the intensive care unit. 3. Indicate the likelihood of a slow recovery. 4. Discuss the likelihood of the child's imminent death.

1 12. 1. Initially, in the ED the child will be given an IV line, vital signs will be taken frequently, and seizure precautions will be taken. 2. The child will be sent to the intensive care unit after being stabilized. 3. When children start to recover from an adrenocortical crisis, their progress is often very rapid, within 24 hours. 4. The child will present with headache, nausea and vomiting, high fever, high blood pres-sure, weakness, and abdominal pain. With prompt diagnosis and the institution of cortisol and fluids, a good recovery is likely.

19. A 13-month-old is discharged following repair of his epispadias. Which statement made by the parents indicates they understand the discharge teaching? 1. "If a mucous plug forms in the urinary drainage tube, we will irrigate it gently to prevent a blockage." 2. "If a mucous plug forms in the urinary drainage tube, we will allow it to pass on its own because this is a sign of healing." 3. "We will make sure the dressing is loosely applied to increase the toddler's comfort." 4. "If we notice any yellow drainage, we will know that everything is healing well."

1 19. 1. Any mucous plugs should be removed by irrigation to prevent blockage of the urinary drainage system. 2. The mucous plug should be removed by irrigation because it could cause a blockage of the urinary drainage system. 3. The dressing is usually a compression type of dressing that helps decrease edema. 4. Foul-smelling yellow drainage is often a sign of infection that needs to be evaluated.

20. A 7-year-old is diagnosed with central precocious puberty. The child is to receive a monthly intramuscular (IM) injection of leuprolide acetate (Lupron). The child has great fear of pain and needles and requires considerable stress reduction techniques each time an injection is due. What could the nurse suggest that might help manage the pain? 1. Apply a eutectic mixture of local anesthetics (EMLA) of lidocaine and prilocaine to the site at least 60 minutes before the injection. 2. Have extra help on hand to help hold the child down. 3. Apply cold to the area prior to injection. 4. Identify a reward to bribe the child to behave during the injection.

1 20. 1. EMLA cream works well for skin and cutaneous pain. Having the child assist in putting on the EMLA patch involves the child in the pain-relieving process. 2. Because this is a monthly injection, it would not be appropriate to hold the child down forcefully, because this creates greater fear and anxiety. 3. Apply ice to the opposite side of the body. 4 Bribing children teaches them that the more they cry and fuss, the more they are rewarded.

23. A child with HUS overy pale and lethargic. Stools have progressed from watery to bloody diarrhea. Blood work indicates low hemoglobin and hematocrit levels. The child has not had any urine output in 24 hours. The nurse expects administration of blood products and what else to be added to the plan of care? 1. Initiation of dialysis. 2. Close observation of the child's hemodynamic status. 3. Diuretic therapy to force urinary output. 4. Monitoring of urinary output.

1 23. 1. Because the child is symptomatic, dialysis is the treatment of choice. 2. Because the child is symptomatic, dialysis is the treatment of choice. 3. Diuretics are given to prevent fluid over-load, but they cannot cause the child in renal failure to produce urine. 4. The nurse would expect the plan to include dialysis because the child is no longer producing urine. Urinary output would still be monitored.

40. The most appropriate nursing diagnosis for a child with type 1 DM is which of the following? 1. Risk for infection related to reduced body defenses. 2. Impaired urinary elimination (enuresis). 3. Risk for injury related to medical treatment. 4. Anticipatory grieving.

1 40. 1. Risk for infection is a correct nursing diagnosis. Understanding DM requires understanding the effect it has on peripheral circulation and impairment of defense mechanisms. 2. Although many children with type 1 DM present with enuresis, impaired urinary elimination is not the best response. 3. Treatment includes injections, but this is not a risk for injury. 4. Type 1 DM, although lifelong, is not a terminal illness and can be well managed, so grieving is not an appropriate diagnosis.

40. Which child is at highest risk for requiring hospitalization to treat RSV? 1. A 2-month-old who was born at 32 weeks. 2. A 16-month-old with a tracheostomy. 3. A 3-year-old with a congenital heart defect. 4. A 4-year-old who was born at 30 weeks.

1 40. 1. The younger the child, the greater the risk for developing complications related to RSV. This infant is at highest risk because of age and premature status. 2. This child has a tracheostomy, but this is not an indication that the child cannot be managed at home. 3. Most children with RSV can be managed at home. Children 2 years and younger are at highest risk for developing complications related to RSV. Children who were born prematurely, have cardiac conditions, or have chronic lung disease are also at higher risk for needing hospitalization. The 3-year-old with a congenital heart disease is not the highest risk among this group of patients. 4. Children who were born prematurely, have cardiac conditions, or have chronic lung disease are at a higher risk for needing hospitalization. This child was a premature infant but is now 4 years of age.

45. The nurse is caring for a child due for surgery on a Wilms tumor. The child's procedure will consist of which of the following? 1. Only the affected kidney will be removed. 2. Both the affected kidney and the other kidney will be removed in case of recurrence. 3. The mass will be removed from the affected kidney. 4. The mass will be removed from the affected kidney, and a biopsy of the tissue of the unaffected kidney will be done.

1 45. 1. The treatment of a Wilms tumor involves removal of the affected kidney. 2. Removal of the unaffected kidney is not necessary and is not done. 3. The entire kidney is removed. 4. A biopsy of the tissue of the unaffected kidney is not necessary and is not obtained.

49. Which is the nurse's best response to the parent of a child diagnosed with epiglottitis who asks what the treatment will be? 1. Complete a course of IV antibiotics. 2. Surgery to remove the tonsils. 3. Ten days of aerosolized ribavirin. 4. No intervention.

1 49. 1. Epiglottitis is bacterial in nature and requires IV antibiotics. A 7- to 10-day course of oral antibiotics is usually ordered following the IV course of antibiotics. 2. Surgery is not the course of treatment for epiglottitis. Epiglottal swelling usually diminishes after 24 hours of intravenous antibiotics. 3. Ribavirin is an antiviral medication used to treat RSV. 4. Epiglottitis is a bacterial infection; a course of IV antibiotics is indicated.

53. The nurse is caring for a 12-year-old receiving peritoneal dialysis. The nurse notes the return to be cloudy, and the child is complaining of abdominal pain. The child's parents ask what the next step will likely be. Which is the nurse's best response? 1. "We will probably place antibiotics in the dialysis fluid before the next dwell time." "Many children experience cloudy returns. We do not usually worry about it." 3. "We will probably give your child some oral antibiotics just to make sure nothing else develops." 4. "The abdominal pain is likely due to the fluid going in too slowly. We will increase the rate of administration with the next fill."

1 53. 1. Cloudy returns and abdominal pain are signs of peritonitis and are usually treated with the administration of antibiotics in the dialysis fluid. 2. Cloudy returns and abdominal pain are signs of peritonitis and need to be treated. 3. Cloudy returns and abdominal pain are signs of peritonitis and are usually treated with the administration of antibiotics in the dialysis fluid. 4. Cloudy returns and abdominal pain are signs of peritonitis. In addition to perito-nitis, abdominal pain can be caused by the rapid infusion of dialysis fluid.

A child receiving peritoneal dialysis is not getting good fluid return. They're edametous and febrile and has HTN. What to do? the health-care provider to do? 1. Increase the glucose concentration of the dialysate. 2. Decrease the glucose concentration of the dialysate. 3. Administer antihypertensives and diuretics but not change the dialysate concentration. 4. Decrease the dwell time of the dialysate.

1 54. 1. Increasing the concentration of glucose will pull more fluid into the return. 2. Decreasing the concentration of glucose will pull less fluid into the return. 3. Antihypertensives and diuretics may be administered, but changing the concentration of glucose in the dialysate will help regulate the fluid balance. 4. Increasing the dwell time would help pull more fluid into the return. Decreasing the dwell time would pull less fluid into the return.

55. The school nurse is talking to a 14-year-old about managing type 1 DM. Which statement indicates the student's understanding of the disease? 1. "It really does not matter what type of carbohydrate I eat as long as I take the right amount of insulin." 2. "I should probably have a snack right after gym class." 3. "I need to cut back on my carbohydrate intake and increase my lean protein intake." 4. "Losing weight will probably help me decrease my need for insulin."

1 55. 1. A carbohydrate is a carbohydrate, and insulin dosing is based on blood sugar level and carbohydrates to be eaten. 2. Snacks should be ingested before planned exercise rather than after. 3. Nutritional needs of children with DM do not differ from those without DM. 4. Weight loss is likely a factor in managing 181 type 2 DM; type 1 DM is often preceded by dramatic weight loss. The nutritional needs of children with type 1 DM are essentially the same as those not affected.

55. During hemodialysis, the nurse notes that a 10-year-old becomes confused and restless. The child complains of a headache and nausea and has generalized muscle twitching. This can be prevented by which of the following? 1. Slowing the rate of solute removal during dialysis. 2. Ensuring the patient is warm during dialysis. 3. Administering antibiotics before dialysis. 4. Obtaining an accurate weight the night before dialysis.

1 55. 1. The child is experiencing signs of disequilibrium syndrome, which is caused by free water shifting from intravascular spaces and can be prevented by slowing the rate of dialysis. 2. The patient's temperature is not a causative factor in disequilibrium syndrome. 3. Antibiotics are used to prevent peritonitis, not disequilibrium syndrome. 4. The child's weight should be obtained immediately prior to dialysis.

56. Chronic hypertension in the child who has CKD is due to which of the following? 1. Retention of sodium and water. 2. Obstruction of the urinary system. 3. Accumulation of waste products in the body. 4. Generalized metabolic alkalosis.

1 56. 1. The retention of sodium and water leads to hypertension. 2. Obstruction of the urinary system can lead to renal failure but is not a direct cause of hypertension. 3. The accumulation of waste products leads to metabolic acidosis. 4. In CKD, the body experiences a state of metabolic acidosis, not alkalosis.

7. Which child is at risk for developing glomerulonephritis? 1. A 3-year-old who had impetigo 1 week ago. 2. A 5-year-old with a history of five UTIs in the previous year. 3. A 6-year-old with new-onset type 1 diabetes. 4. A 10-year-old recovering from viral pneumonia.

1 7. 1. Impetigo is a skin infection caused by the streptococcal organism that is commonly associated with glomerulonephritis. 2. Frequent UTIs have not been associated with glomerulonephritis. 3. Type 1 diabetes is not a cause of glomerulonephritis. 4. Glomerulonephritis can be caused by a streptococcal organism, not a viral pneumonia.

8. A teen who was hospitalized for CKD develops symptoms of polyuria, polydipsia, and bone pain. Which body mineral might be causing these symptoms? 1. Elevated calcium, 2. Low phosphorus. 3. Low magnesium. 4. High aluminum hydroxide.

1 8. 1. The most common causes of secondary hyperparathyroidism are chronic renal disease and anomalies of the urinary tract. Blood studies indicate very high levels of calcium because the kidney is unable to process it. 2. Renal impairment causes phosphorus levels to become very high, and clients with CKD are often put on low-phosphorus diets to control the levels. 3. Magnesium levels are elevated in CKD because magnesium is excreted in the urine. Magnesium is not regulated by the parathyroid gland. 4, Aluminum hydroxide keeps phosphorus mobilized so that it can be excreted.

12. Which statement by a parent is most consistent with MCNS? 1. "My child missed 2 days of school last week because of a really bad cold." 2. "After camping last week, my child's legs were covered in bug bites." 3. "My child came home from school a week ago because of vomiting and stomach cramps." 4. "We have a pet turtle, but no one washes their hands after playing with the turtle

1 12. 1. An upper respiratory infection often precedes MCNS by a few days. 2. Bug bites are not typically associated with MCNS. 3. Vomiting and abdominal cramping are not typically associated with MCNS. 4. Pet turtles often carry salmonella, which can cause vomiting and diarrhea but not MCNS.

37. An 8-year-old with type 1 DM is complaining of a headache and dizziness and is visibly perspiring. Which of the following should the nurse do first? 1. Administer glucagon intramuscularly. 2. Offer the child 8 oz of milk. 3. Administer rapid-acting insulin lispro (Humalog): 4. Offer the child 8 oz of water or calorie-free liquid.

2 37. 1. Glucagon is given only for severe hypogly-cemia. The child's symptoms are those of mild hypoglycemia. 2. Milk is best to give for mild hypogly-cemia, which would present with the symptoms described. 3. Insulin is appropriate for elevated blood glucose, but the symptoms listed are those of hypoglycemia, not hyperglycemia. It is important for the test taker to be able to distinguish between the two. 4. Water is appropriate for mild hyperglyce-mia, but the symptoms listed are those of hypoglycemia.

13. A child with Addison disease takes oral cortisol supplements. What teaching should be done at each visit? Select all that apply. 1. "Keep an extra month's supply of all medications on hand at all times." 2. "Wear a MedicAlert bracelet at all times." "The child is less susceptible to infections." 4. "Weight gain will occur." "Your child is more susceptible to infections."

13. 1, 5. Keeping an extra month's supply of all medications, along with a prefilled syringe of hydrocortisone, will enable the family to treat an impending adrenal crisis before it becomes severe. 2. Wearing a MedicAlert bracelet is advantageous and can be periodically mentioned to the family. 3. Children are more susceptible to infections when on a chronic anti-inflammatory medication such as cortisol. 4. Weight gain will occur over time, but with good diet and adequate exercise, it can be controlled. 5. Cortisol is an anti-inflammatory drug; with long-term use, it causes the child to be more susceptible to infections.

21. A child weighs 21 kg. The parent asks for the weight in pounds. Which is the correct equivalent? 1. 9.5 lb. 2. 46.2 lb oz. 3. 50 lb. 4. 60 lb.

2 21. 1. 2.2 lb = 1 kg. Multiplying the kilograms by 2.2 yields 46.2 lb. 2. 2.2 Ib = 1 kg. Multiplying the kilograms by 2.2 yields 46.2 lb. 3. 2.2 1b = 1 kg. Multiplying the kilograms by 2.2 yields 46.2 lb. 4. 2.2 lb = 1 kg. Multiplying the kilograms by 2.2 yields 46.2 lb.

22. Parents bring their teen to the clinic with a tender, enlarged right breast. The nurse explains that which hormone(s) secreted by the anterior pituitary influence(s) this process? 1. Thyrotropin. 2. Gonadotropin. 3. Oxytocin. 4. Somatotropin.

2 22. 1. Thyrotropin and somatotropin are secreted by the anterior pituitary but do not directly influence breast development. 2. The gonadotropins stimulate the gonads to mature and produce sex hormones. 3. Oxytocin is secreted by the posterior pituitary and targets the uterus and production of milk from the breasts. 4. Thyrotropin and somatotropin are secreted by the anterior pituitary but do not directly influence breast development.

The nurse is providing discharge instructions for an infant born with bladdar extrophy who has a continent urinary reservoir placed which statement should be included? .1. "Allow your child to sleep on the abdomen to provide comfort during the immediate postoperative period." 2. "As your child grows, be cautious around playgrounds because the surface could be a health hazard." 3. "As your child grows, be sure to encourage many different foods because it is not likely that food allergies will develop." 4. "Encourage your child's development by having brightly cooored objects around like baloons

2 28. 1. The infant should not be allowed to sleep on the abdomen because the prone position has been associated with sudden infant death syndrome. 2. Many children with urological malformations are prone to latex allergies. The surfaces of playgrounds are often made of rubber, which contains latex. 3. Many children with urological malformations are prone to latex allergies. Foods such as bananas can cause a latex allergy. 4. Although children need a stimulating envi-ronment, balloons are dangerous because many contain latex and can also be a choking hazard.

29. A parent with a toddler who has ambiguous genitalia asks the nurse how long it will be before the child identifies his or her gender. Which is the best answer? 1. "Children do not know their gender until they are a teen." 2. "Children know their gender by the age of 18 to 30 months." 3. "Children know from the time of birth what their gender is." 4. "Children of 4 to 6 years are beginning to learn their gender."

2 29. 1. Children know their gender well before they are teens and may try on the roles of the opposite gender. 2. Children 18 months to 30 months learn their gender from examining and touching their body parts and learning gender roles from caregivers and others 3. Newborns do not identify with one gender or the other. 4. By 4 to 6 years of age, children have a clear understanding of their gender and whether it aligns with their gender assigned at birth.

3. A 6-year-old white girl comes with her mother for evaluation of her acne, breast buds, axillary hair, and body odor. What information should the nurse explain to them? 1. This is a typical age for girls to go into puberty. 2. Encourage the girl to dress and act appropriately for her chronological age. 3. She should be on birth control because she is fertile. 4. She may be short if her epiphyses close early.

2 3. 1. Although girls in the United States mature earlier than in previous decades, the lowest age at which puberty is considered normal is 7 years for white girls and 6 years for African American girls. 2. Dressing and acting appropriately for her chronological age should be encouraged for the well-being of the child. 3. Although she is fertile if she is pubescent, it Is not developmentally necessary to consider the use of birth control at this time. 4. The estrogen that is produced during puberty does assist in the closure of the epiphyseal plates. With proper medication management, however, the estrogen will be suppressed, and she will continue to grow.

31. The parent brings the growth record along with the 21-month-old child to a new clinic for a well-child visit. The record shows a birth weight of 8 Ib; the 6-month weight was 16 lb; the 12-month weight was 18 lb; and the 15-month weight was 19 Ib. With the record showing that the toddler's weight-for-age has been decreasing, the nurse should do what initially? 1. Omit plotting the previous weight-for-age on the new growth chart. 2. Point out the growth chart to the new HCP. 3. Consider the toddler a child with failure to thrive. 4. Weigh the child and plot on a new growth chart.

2 31. 1. The data from the previous primary care provider are important for comparison purposes. 2. The provider should be made aware of the decelerating weight for age. This pictorial information can then be reviewed with the parent. 3. Failure to thrive is a diagnosis that can be organic, nonorganic, or idiopathic. Labeling a patient as having failure to thrive prior to the assessment is inappropriate. 4. Weighing toddlers can be a challenge because they are very active. The new weight should be obtained with only a diaper on the child. It should be recorded along with the previous provider's weights.

32. The nurse in a diabetic clinic sees a 10-year-old who is newly diagnosed with diabetes and has had trouble maintaining blood glucose levels within normal limits. The child's parent states the child has had several daytime "accidents." The nurse knows that this is referred to as which of the following? 1. Primary enuresis. 2. Secondary enuresis. 3. Diurnal enuresis. 4. Nocturnal enuresis.

2 32. 1. Primary enuresis refers to urinary incontinence in a child who has never had voluntary bladder control. 2. Secondary enuresis refers to urinary incontinence in a child who previously had bladder control. 3. Diurnal enuresis refers to daytime urinary incontinence not caused by something else. 4. Nocturnal enuresis refers to nighttime urinary incontinence.

school-age child comes in with a sore throat and fever. The child was recently diagnosed with Graves disease and is taking propylthiouracil. What concerns should the nurse have about this child? 1. The child must not be taking her medication. 2. The child may have leukopenia. 3. The child needs to start an antibiotic. 4. The child is not getting enough sleep.

2 36. 1. Propylthiouracil is used to suppress thyroid function. One of the grave complications of the medication is leukopenia. 2. Propylthiouracil is used to suppress thyroid function. One of the grave complications of the medication is leukopenia. 3. Propylthiouracil is used to suppress thyroid function. One of the grave complications of the medication is leukopenia. 4. Propylthiouracil is used to suppress thyroid function. One of the grave complications of the medication is leukopenia.

38. The nurse is caring for a 10-year-old post parathyroidectomy. Discharge teaching should include which of the following? 1. How to administer injectable growth hormone. 2. The importance of supplemental calcium in the diet. 3. The importance of increasing iodine in the diet. 4. How to administer subcutaneous insulin.

2 38. 1. Growth hormone would be appropriate if the patient had panhypopituitarism. 2. The parathyroid is responsible for calcium reabsorption; therefore, supplemental calcium in the diet is the important point to be discussed in patient teaching. 3. Iodine is another important dietary supplement. 4. Information about insulin injection is appropriate for diabetes mellitus, not parathyroid disorder.

38. The parents of a 6-week-old male client ask the nurse if there is a difference between an inguinal hernia and a hydrocele. Which is the nurse's best response? 1. "The terms are used interchangeably and mean the same thing." 2. "The symptoms are similar, but an inguinal hernia occurs when tissue protrudes into the groin, whereas a hydrocele is a fluid-filled mass in the scrotum." 3. "Hydrocele' is the term used when an inguinal hernia occurs in females." 4. "A hydrocele presents in a manner similar to that of an inguinal hernia but causes increased concern because it is often malignant."

2 38. 1. The terms are not used interchangeably. "Inguinal hernia" refers to protrusion of abdominal tissue into the groin, and "hydrocele" refers to a fluid-filled mass in the scrotum. 2. The symptoms are similar, but an inguinal hernia occurs when tissue protrudes into the groin; a hydrocele is a fluid-filled mass in the scrotum. 3. A hydrocele does not occur in females. 4. A hydrocele is not associated with an increased risk of malignancy.

43. The nurse is obtaining the medical history of an 11-year-old diagnosed with hypo-pituitarism. An important question for the nurse to ask the parents is which of the following? 1. "Is the child receiving vasopressin intramuscularly or subcutaneously?" 2. "What time of day do you administer growth hormone?" 3. "Does your child have any concerns about being taller than the peer group?" 4. "How often is your child testing blood glucose?"

2 43. 1. Vasopressin is a medication used to treat the uncontrolled diuresis associated with DI. 2. Growth hormone is used to treat a child with hypopituitarism. 3. Hypopituitarism promotes short, not tall, stature. 4. Although hypoglycemia might be a factor when a child is undergoing diagnostic testing for hypopituitarism, blood sugar testing is not part of the treatment plan.

46. The nurse anticipates that the child who has had a kidney removed will have a high level of pain and will require invasive and noninvasive measures for pain relief. The nurse anticipates that the child will have pain because of which of the following? 1. The kidney is removed laparoscopically, and there will be residual pain from accumulated air in the abdomen. 2. There is a postoperative shift of fluids and organs in the abdominal cavity, leading to increased discomfort. 3. The chemotherapy makes the child more sensitive to pain. 4. The radiation therapy makes the child less sensitive to pain.

2 46. 1. A large incision is used because the kidney is not removed laparoscopically at this time. 2. There is a postoperative shift of fluids and organs in the abdominal cavity, leading to increased discomfort. 3. The increased pain is due to shifting of fluid and organs. 4. The increased pain is due to shifting of fluid and organs.

50. The nurse is taking care of a 10-year-old diagnosed with Graves disease. Which could the nurse expect this child to have recently had? 1. Weight gain, excessive thirst, and excessive hunger. 2. Weight loss, difficulty sleeping, and heat sensitivity. 3. Weight gain, lethargy, and goiter. 4. Weight loss, poor skin turgor, and constipation.

2 50. 1. Graves disease is a type of hyperthyroid-ism. Gradual weight loss, not weight gain, is a sign. 2. Weight loss, increased activity, and heat intolerance can be expected when the thyroid gland is hyperfunctional. 3. Weight gain as a symptom makes this answer incorrect. 4. Constipation and dry skin with poor turgor are more likely in a patient with hypothyroidism.

52. A 5-year-old is brought to the ED with a temperature of 99.5°F (37.5°C), a barky cough, stridor, and hoarsenes. Which nursing intervention should the nurse prepare for? 1. Immediate IV placement. 2 Respiratory treatment of racemic epinephrine. 3. A tracheostomy set at the bedside. 4. Inform the parents about the vital signs.

2 52. 1. The child is exhibiting signs and symptoms of croup and is in mild respiratory distress. 2. The child has stridor, indicating airway edema, which can be relieved by aerosolized racemic epinephrine. 3. A tracheostomy is not indicated for this child. A tracheostomy would be indicated for a child with a complete airway obstruction. 4. This child is exhibiting signs and symptoms of croup. The nurse will quickly give the parents information about the vital signs, then get a treatment for the child.

54. Which is the most likely reason an adolescent with DM has problems with low self-esteem? 1. Managing diabetes decreases independence. 2. Managing diabetes complicates perceived ability to "fit in." 3. Obesity complicates perceived ability to "fit in.' 4. Hormonal changes are exacerbated by fluctuations in insulin levels.

2 54. 1. Managing DM increases rather than decreases independence. 2. Because the desire to fit in is so strong in adolescence, the need to manage one's diabetes can compromise the patient's perception of ability to do so. For example, an adolescent with type 1 DM has to plan meals and snacks, test blood sugar, limit choices of when and what to eat, and always be concerned with the immediate health consequences of actions as simple as eating. The fact that these limitations can negatively affect self-esteem is an essential concept for the nurse caring for adolescents with diabetes to understand. 3. Obesity is not necessarily associated with type 1 DM. 4. Hormone changes in adolescence often "esult in increasing insulin demands; how-ever, increased insulin need not correlate with low self-esteem.

54. Which statement about pneumonia is accurate? 1. Pneumonia is most frequently caused by bacterial agents. 2. Children with bacterial pneumonia are usually sicker than children with viral pneumonia. 3. Children with viral pneumonia are usually sicker than those with bacterial pneumonia. 4. Children with viral pneumonia must be treated with a complete course of antibiotics.

2 54. 1. Pneumonia is most frequently caused by viruses but can also be caused by bacteria such as Streptococcus pneumonae. 2. Children with bacterial pneumonia are usually sicker than children with viral pneumonia. Children with bacterial pneumonia can be treated effectively, but they require a course of antibiotics. 3. Children with viral pneumonia are not usually as ill as those with bacterial pneu-monia. Treatment for viral pneumonia includes maintaining adequate oxygenation and comfort measures. 4. Treatment for viral pneumonia includes maintaining adequate oxygenation and comfort measures.

56. The nurse is assigned to care for a newborn with goiter. The nurse's primary concern is which of the following? 1. Reassuring the parents that the condition is only temporary and will be treated with medication. 2. Maintaining a patent airway and preparing for emergency ventilation. 3. Preparing the infant for surgery and initiating preoperative teaching with the parents. 4. Obtaining a detailed history, particularly of medications taken during the mother's pregnancy.

2 56. 1. Goiter in a newborn can be life threatening and is usually treated by surgical removal or partial removal of the thyroid. 2. Goiter in a newborn can cause tracheal compression, and positioning to help relieve pressure (i.e., neck hyperex-tension) is essential. Emergency precautions for ventilation and possible tracheostomy are also instituted. 3. Although preparation for surgery might be necessary, the most important intervention is protecting the infant's airway. 4. Certain medications (antithyroid drugs) taken by the mother could predispose the infant to developing a goiter. Although the history is important, it is not the first priority for the nurse caring for this patient.

56. Which position would be most comfortable for a child with left-sided pneumonia? 1. Trendelenburg. 2. Left side. 3. Right side. 4. Supine.

2 56. 1. The Trendelenburg position is not effective for improving respiratory effort. Patients with pneumonia are usually most comfortable in a semi-erect position. 2. Lying on the left side may provide the patient with the most comfort. Lying on the left splints the chest and reduces the pleural rubbing. 3. It is most comfortable for the patient to lie on the affected side. Lying on the left splints the chest and reduces the pleural rubbing. 4. Lying in the supine position does not provide comfort for the patient and does not improve the child's respiratory effort.

58. A 3-year-old is brought to the ED with coughing and gagging. The parent reports that the child was eating carrots when she began to gag. Which diagnostic evaluation will be used to determine whether the child has aspirated carrots? 1. Chest x-ray. 2. Bronchoscopy. 3. ABG. 4. Sputum culture.

2 58. 1. A chest x-ray will show only radiopaque items (items that x-rays cannot go through easily), so it is not helpful in determining whether the child aspirated a carrot. 2. A bronchoscopy will allow the physician to visualize the airway and will help determine whether the child aspirated the carrot. 3. A blood gas will identify whether the child has suffered any respiratory compromise, but the ABG cannot definitively determine the cause of the compromise. 4. A sputum culture may be helpful several days later to determine whether the child has developed aspiration pneumonia. Aspiration pneumonia may take several days or a week to develop following aspiration.

6. The school nurse notices that a 14-year-old who used to be an excellent student and very active in sports is losing weight and acting very nervous. The teen was recently checked by the PCP, who noted the teen had a very low level of TSH. The nurse recognizes that the teen has which condition? 1. Hashimoto thyroid disease. 2. Graves disease. 3. Hypothyroidism. 4. Juvenile autoimmune thyroiditis.

2 6. 1. Hashimoto thyroiditis is a hypothyroid dis-ease. Laboratory tests would reveal a high TSH level. 2. Graves disease is hyperthyroidism and presents with low TSH levels, weight loss, and excessive nervousness. 3. Hypothyroidism is accompanied by a high TSH level. 4. Juvenile autoimmune thyroiditis is a hypothyroid disease. Laboratory tests would reveal a high TSH level.

7. A newborn develops tetany and has a seizure prior to discharge from the nursery. The newborn is diagnosed with hypocalcemia secondary to hypoparathyroidism and is started on calcium and vitamin D. Which information would be most important for the nurse to teach the parents? 1. They should observe the baby for signs of tetany and seizures. L. They should observe for weakness, nausea, vomiting, and diarrhea. 3. They should administer the calcium and vitamin D daily as prescribed. 4. They should call the clinic if they have any questions about care of the newborn.

2 7. 1. The baby has hypocalcemia and is being treated for this condition by the team. This should be reviewed with the family. 2. Vitamin D toxicity (weakness, nausea, vomiting, and diarrhea) is a serious consequence of therapy and should be the top priority in teaching. 3. Reminding the family to give the medication as prescribed is helpful and should be a basic part of discharge care, but it is not the most important information. 4. Giving the family the phone number for calling the clinic is part of basic care for discharge to home, but it is not the most important information.

25. The adrenal corter secretes sex hormones. Identify which hormones would result in feminization of a young male child. Select all that apply. 1. Estrogen. 2. Testosterone. 3. Progesterone. 4. Cortisol. 5. Androgens.

25. 1, 3. 1. Estrogen and progesterone are hormones secreted by the adrenal cortex that in excess would cause feminization of a young male child. 2. Testosterone is secreted by the testes and does not result in feminization. 3. Estrogen and progesterone are hormones secreted by the adrenal cortex that in excess would cause feminization. 4. Cortisol is a glucocorticoid that is secreted by the adrenal cortex but does not result in feminization. 5. Androgens are secreted by the adrenal cortex but would cause masculinization of the child.

1. At a follow-up visit for an 8-year-old who is being evaluated for short stature, the nurse measures and plots the child's height on the growth chart. Which explanation should the nurse give the child and family? 1. "We want to make sure you were measured accurately the last two visits." "We need to calculate how tall you will be when you grow to adult height." "We need to see how much you have grown since your last visit." 4. "We need to know your height so that a dosage of medication can be calculated for you."

3 1. 1. Height velocity is important to determine, not whether the measurement was incorrect at a previous visit. 2. Expected adult height can be determined using a formula that takes into account the parents' height and can be calculated at any visit. 3. Height velocity is the most important aspect of a growth evaluation and can demonstrate deceleration in growth if it is present. 4. The dose of growth hormone replacement medication is based on weight.

Which finding requires immediate attention in a child with glomerulonephritis? 1. Sleeping most of the day and being very cranky when awake; blood pressure is 170/90. 2. Urine output is 190 mL in an 8-hour period and is the color of cola. 3. Complaining of a severe headache and photophobia. 4. Refusing breakfast and lunch and stating he is "just not hungry."

3 10. 1. Children with glomerulonephritis usually have an elevated blood pressure and tend to rest most of the day. 2. The urine output is often decreased, and the urine is often tea-colored due to hematuria. 3. A severe headache and photophobia can be signs of encephalopathy due to hypertension, and the child needs immediate attention. 4. Anorexia is often seen with glomerulonephritis.

15. A child with MCNS has generalized edema. The skin appears stretched, and areas of breakdown are noted over the bony prominences. The child has been receiving furosemide (Lasix) twice daily for several days. Which does the nurse expect to be included in the treatment plan to reduce edema? 1. An increase in the amount and frequency of furosemide. 2. Addition of a second diuretic, such as mannitol. 3. Administration of IV albumin. 4. Elimination of all fluids and sodium from the child's diet

3 15. 1. The dosage of the diuretic may be adjusted, but other medications, such as albumin, are likely to be used. 2. Mannitol is not usually used in the treatment of MCNS. 3. In cases of severe edema, albumin is used to help return the fluid to the bloodstream from the subcutaneous tissue. 4. Although sodium and fluids are restricted in the severely edematous child, they are not eliminated completely.

17. The nurse is caring for a newborn with hypospadias. His parents ask if circumcision is an option. Which is the nurse's best response? 1. "Circumcision is a fading practice and is now contraindicated in most children." 2. "Circumcision in children with hypospadias is recommended because it helps prevent infection." 3. "Circumcision is an option, but it cannot be done at this time." 4. "Circumcision can never be performed in a child with hypospadias."

3 17. 1. Routine circumcision is recommended by the American Academy of Pediatrics; it is not contraindicated in most children. 2. It is not recommended that circumcision of children with hypospadias be done immediately because the foreskin may be needed later for repair of the defect. 3. It is usually recommended that circumcision be delayed in the child with hypospadias because the foreskin may be needed for repair of the defect. 4. Circumcision can usually be performed in the child with hypospadias when the defect is corrected.

18. An infant is scheduled for a hypospadias and chordee repair. The parent asks the nurse, "I understand why the hypospadias repair is necessary, but do they have to fix the chordee as well?" Which is the nurse's best response? 1. "I understand your concern. Parents do not want their children to undergo extra surgery." 2. "The chordee repair is done strictly for cosmetic reasons that may affect your son as he ages." 3. "The repair is done to optimize sexual functioning when he is older." "This is the best time to repair the chordee because he will be having surgery anyway."

3 18. 1. This response is empathetic. It does not, however, answer the parent's concern, whereas a simple explanation would immediately do so. 2. Although a cosmetic component exists, straightening the penis is important for 3. Releasing the chordee surgically is necessary for future sexual function 4. Although the two surgeries are usually done simultaneously, each has its own importance and necessity.

2. The nurse is caring for a 4-year-old who weighs 15 kg. At the end of a 10-hour period, the nurse notes the urine output to be 150 mL. What action does the nurse take? 1. Notifies the health-care provider because this urine output is too low. 2. Encourages the child to increase oral intake to increase urine output. 3. Records the child's urine output in the chart. 4. Administers isotonic fluid intravenously to help with rehydration.

3 2. 1. The child weighs 15 kg, and the expected urine output is 0.5 to 1 mL/kg/hr. 0.5 × 15 kg = 7.5 mL; 1 mL × 15 kg = 15 mL. 7.5-15 mL/hour x 10 hours = 75-150 mL of urine for the 10-hour period. Therefore, the output is not too low. 2. The child weighs 15 kg, and the expected urine output is 0.5-1 mL/kg/hr. 0.5 × 15 kg = 7.5 mL; 1 mL x 15 kg = 15 mL. 7.5-15 mL/hour × 10 hours = 75-150 mL of urine for the 10-hour period. The urine output is not too low, so the nurse does not need to encourage more fluids. 3. Recording the child's urine output in the chart is the appropriate action because the urine output is within the expected range of 0.5 to 1 mL/kg/hr, or 75 to 150 mL for the 10-hour period. 4. The child weighs 15 kg and the expected urine output is 0.5 to 1 mL/kg/hr. 0.5 × 15 kg = 7.5 mL; 1 mL × 15 kg = 15 mL. 7.5-15 mL/hour x 10 hours = 75-150 mL of urine for the 10-hour period. Therefore, it is not too low. The child is hydrated.

25. A 5-year-old is discharged from the hospital following the diagnosis of HUS. The child has been free of diarrhea for 1 week, and renal function has returned. The parent asks the nurse when the child can return to school. Which is the nurse's best response? 1. "Immediately, as your child is no longer contagious." 2. "It would be best to keep your child home for a few more weeks because the immune system is weak, and there could be a relapse of HUS." "Your child will be contagious for approximately another 10 days, so it is best to not allow a return just yet." 4. "It would be best to keep your child home to monitor urinary output."

3 25. 1. Children with HUS are considered contagious for up to 17 days after the resolution of diarrhea and should be placed on contact isolation. 2. Once the child recovers from HUS, there is usually no relapse. 3. Children with HUS are considered contagious for up to 17 days after the resolution of diarrhea and should be placed on contact isolation. 4. Once free of diarrhea for approximately 17 days, the child is considered not to be contagious and should be encouraged to return to developmentally appropriate activities as tolerated.

The parent of a 7-year-old voices concern over the child's continued bed-wetting at night. The parent, on going to bed, has tried getting the child up at 11:30 p.m., but the child still wakes up wet. Which is the nurse's best response about what the parent should do next? 1. "There is a medication called DDAVP that decreases the volume of the urine. The physician thinks that will work for your child." 2. "When your child wakes up wet, be very firm and indicate how displeased you are. Have your child change the sheets to see how much work is involved." 3. "Limit fluids in the evening and start a reward system in which your child can choose a reward after a certain number of dry nights." 4. "Bed-wetting alarms are readily available, and most children do very well with them."

3 33. 1. Although DDAVP is used for enuresis, it is not the first treatment chosen. Behavior modification and positive reinforcement are usually tried first. 2. Having the child help with changing the bed is a good idea. The child should be approached in a positive manner, however, not a punitive one, so as not to threaten 3. Limiting the child's fluids in the evening will help decrease the nocturnal urge to void. Providing positive reinforcement and allowing the child to choose a reward will increase the child's sense of control. 4. Enuresis alarms are readily available, but behavior modification and positive reinforcement are usually tried first.

33. A child with adrenal insufficiency is sick with influenza. The parent calls the office and wants to know what to do. What is the first thing the nurse should advise this parent? 1. Withhold all medications and bring the child to the office. 2. Encourage the child to drink water and juices. 3. Give the child a dose of hydrocortisone and bring the child to the office. 4. Let the child rest; the child will be better in the morning.

3 33. 1. The family should administer the reserve hydrocortisone injection intramuscularly immediately and then bring the child to the office. 2. When children with adrenal insufficiency get influenza or are ill, the situation can be life threatening. The family should administer the reserve hydrocortisone injection intramuscu-larly and then bring the child to the office. 3. When children with adrenal insufficiency get influenza or are ill, the situation can be life threatening. The family should administer the reserve hydrocortisone injection intramuscularly and then bring the child to the office. There is no harm in giving extra hydrocortisone. 4. When children with adrenal insufficiency get influenza or are ill, the situation can be life threatening.

42. Which would the nurse expect to find on assessment in a child with Wilms tumor? 1. Decreased blood pressure, increased temperature, and a firm mass located in one flank area. 2. Increased blood pressure, normal temperature, and a firm mass located in one flank area. 3. Increased blood pressure, normal temperature, and a firm mass located on one side of the midline of the abdomen. 4. Decreased blood pressure, normal temperature, and a firm mass located on one side or the other of the midline of the abdomen.

3 42. 1. The blood pressure may be increased, not decreased, if there is renal damage. The mass will be located on one side or the other of the midline of the abdomen. There is no reason for the child's temperature to be affected 2. The blood pressure may be increased if there is renal damage. The mass will be located on one side or the other of the midline of the abdomen. There is no reason for the child's temperature to be affected. 3. The blood pressure may be increased if there is renal damage. The mass will be located on one side or the other of the midline of the abdomen. There is no reason for the child's temperature to be affected. 4. The blood pressure may be increased, not decreased, if there is renal damage. The mass is located on one side or the other of the midline of the abdomen. There is no reason for the child's temperature to be affected.

43. A child diagnosed with a Wilms tumor is scheduled for an MRI scan of the lungs. The parent asks the nurse the reason for this test, as a Wilms tumor involves the kidney, not the lung. Which is the nurse's best response? 1. "T'm not sure why your child is going for this test. I will check and get back to you." 2. "It sounds like we made a mistake. I will check and get back to you." "The test is done to check whether the disease has spread to the lungs." 4. "We want to check the lungs to make sure your child is healthy enough to tolerate surgery."

3 43. 1. When the nurse is unsure of the answer, it is best to check and get back to the parents. The nurse should be aware that tests of other organs are often performed to evaluate for the presence of metastases. 2. The test is ordered to check for metastasis to the lungs. 3. The test is done to see if the disease has spread to the lungs. 4. A chest x-ray, not a magnetic resonance image, is ordered routinely to evaluate the health of the lungs prior to surgery.

44. Which is an important nursing intervention for a child with a diagnosis of hyperthyroidism? 1. Encourage an increase in physical activity. 2. Do preoperative teaching for thyroidectomy. 3. Promote opportunities for periods of rest. 4. Do dietary planning to increase caloric intake.

3 44. 1. A child with hyperthyroidism displays increased activity. 2. Thyroidectomy is not the first choice for treating hyperthyroidism, although it is considered if other treatments fail. 3. Because increased activity is characteristic of hyperthyroidism, providing opportunity for rest is a recommended nursing intervention. 4. Weight loss despite increased appetite and intake might be a symptom of hyperthy-roidism. Just increasing calories is not the best option because the cause and treatment of the thyroid's hyperactivity needs to be addressed.

45. A 13-year-old with type 2 DM asks the nurse, "Why do I need to have this hemoglobin Alc test?" The nurse's response is based on which of the following? 1. To determine how balanced the child's diet has been. 2. To make sure the child is not anemic. 3. To determine how controlled the child's blood sugar has been. 4. To make sure the child's blood ketone level is normal.

3 45. 1. Balanced diet, although important, is not determined by hemoglobin Alc. 2. Anemia would be a correct choice if the question asked about hemoglobin, not hemoglobin Alc. 3. Hemoglobin Alc, or glycosylated hemoglobin, reflects average blood glucose levels over 2 to 3 months. Frequent high blood glucose levels would result in a higher hemoglobin Alc, suggesting that blood glucose needs to be in better control. 4. Presence of ketones in the blood, although associated with the absence of insulin and with high blood glucose levels, is not directly correlated with hemoglobin Alc.

46. The nurse caring for a 14-year-old girl with DI understands which of the following about this disorder? 1. DI is treated on a short-term basis with hormone replacement therapy. 2. DI may cause anorexia if proper meal planning is not addressed. 3. DI is treated with vasopressin on a lifelong basis. 4. DI requires strict fluid limitation until it resolves.

3 46. 1. DI is caused by undersecretion of ADH (vasopressin). Replacement therapy with vasopressin is a long-term option, however, not a short-term one. 2. DI is not to be confused with DM, for which dietary planning is a large part of management. 3. Vasopressin is the treatment of choice. It is important for patients and parents to understand that DI is a lifelong disease. 4. Although a sign of DI is excessive uri-nation, fluid replacement is needed, not restriction. Fluid restriction is only part of the diagnostic phase.

47. Which is diagnostic for epiglottitis? 1. Blood test. 2. Throat swab. 3. Lateral neck x-ray of the soft tissue. 4. Signs and symptoms.

3 47. 1. A blood test does not provide a diagnosis of epiglottitis. A CBC may show an increased white blood cell count, indicating the child has some sort of infection. 2. A throat culture is not done to diagnose epiglottitis. It is contraindicated to insert anything into the mouth or throat of any child who is suspected of having epiglotti-tis. Inserting anything into the throat could cause the child to have a complete airway obstruction. 3. A lateral neck x-ray is a definitive test to diagnose epiglottitis. The child is at risk for complete airway obstruction and should always be accompanied by a nurse to the x-ray department. 4. Epiglottitis is not diagnosed on the basis of signs and symptoms. A lateral neck film makes the diagnosis.

51. A 12-year-old with type 2 DM presents with a fever and a 2-day history of vomiting. The nurse observes that the child's breath has a fruity odor and breathing is deep and rapid. Which should the nurse do first? 1. Offer the child 8 oz of clear noncaloric fluid. 2. Test the child's urine for ketones. 3. Prepare the child for an IV infusion. 4. Offer the child 25 g of carbohydrates.

3 51. 1. The fruity odor is that of acetone. The patient is exhibiting signs of ketoacidosis. The history of vomiting and the Kussmaul breathing preclude oral rehydration. 2. Although it is likely that ketones would be present, the child is in a life-threatening situation. Checking urine is not necessar 3. This patient needs fluid and electrolyt. therapy to restore tissue perfusion prior to beginning IV insulin therapy. 4. The patient is hyperglycemic, not hypoglycemic.

52. Which would the school nurse expect in a student who has an insulin-to-carbohydrate ratio of 1:10? 1. The student administers 10 U of regular insulin for every gram of carbohydrate consumed. 2. The student is trying to limit carbohydrate intake to 10 g per insulin dose. 3. The student administers 1 U of regular insulin for every 10 grams of carbohydrate consumed. 4. The student plans to eat 10 g of carbohydrate for every dose of insulin.

3 52. 1. An insulin-to-carbohydrate ratio refers to the amount of insulin given per gram of carbohydrate. A ratio of 1:10 means 1 U regular insulin for every 10 g carbohy-drates. This dose would be appropriate if the child were planning a meal of 100 g of carbohydrates. 2. Limiting carbohydrate intake is not a factor in managing type 1 DM. 3. An insulin-to-carbohydrate ratio refers to the amount of insulin given per gram of carbohydrate. A ratio of 1:10 means 1 U regular insulin for every 10 g carbohydrates. 4. An insulin-to-carbohydrate ratio refers to the amount of insulin given per gram of carbohydrate. A ratio of 1:10 means 1 U regular insulin for every 10 g carbohydrates.

52. The parent of a child diagnosed with AKI asks the nurse why peritoneal dialysis was selected instead of hemodialysis. Which is the nurse's best response? 1. "Hemodialysis is not used in the pediatric population." 2. "Peritoneal dialysis has no complications, so it is a treatment used without hesitation." 3. "Peritoneal dialysis removes fluid at a slower rate than hemodialysis, so many complications are avoided." 4. "Peritoneal dialysis is much more efficient than hemodialysis."

3 52. 1. Hemodialysis is used in the pediatric population. Peritoneal dialysis has many complications, such as peritonitis. 3. Peritoneal dialysis removes fluid at a slower rate that is more easily controlled than that of hemodialysis. 4. Hemodialysis is much more efficient than peritoneal dialysis.

55. Which child diagnosed with pneumonia would benefit most from hospitalization? 1. A 13-year-old who is coughing, has coarse breath sounds, and is not sleeping well. 2. A 14-year-old with a fever of 38.6°C (101.5°F), rapid breathing, and a decreased appetite. 3. A 15-year-old who has been vomiting for 3 days and has a fever of 38.5°C (101.3°F). 4. A 16-year-old who has a cough, chills, fever of 38.5°C (101.3°F), and wheezing.

3 55, 1. These are all common symptoms of pneumonia and should be monitored but do not require hospitalization. Most people with pneumonia are treated at home, with a focus on treating the symptoms and keeping the patient comfortable, Comfort measures include cool mist, CPT, antipyretics, fluid intake, and family support. 2. These are all common symptoms of pneumonia and should be monitored but do not require hospitalization. 3. The teen who has been vomiting for several days and is unable to tolerate oral fluids and medication should be admitted for IV hydration. 4. These are all common symptoms of pneumonia and should be monitored but do not require hospitalization.

34. A toddler is being evaluated for SIADH. The nurse should observe the child for which symptoms? Select all that apply. 1. Dehydration. 2. Fluid retention. 3. Hyponatremia. 4. Hypoglycemia. 5. Myxedema.

34. 2, 3. 1. Dehydration is not a feature of SIADH. 2. ADH assists the body in retaining fluids and subsequently decreases serum osmolarity while the urine osmolarity rises. When serum sodium levels are decreased below 120 mEg/L, the child becomes symptomatic. 3. ADH assists the body in retaining fluids and subsequently decreases serum osmolarity while the urine osmolarity rises. When serum sodium levels are decreased below 120 mEg/L, the child becomes symptomatic. 4. Hypoglycemia is not a feature of SIADH. 5. Myxedema is not a feature of SIADH.

57. A 13-year-old is being seen for an annual physical examination. The child has lost 10 lb despite reports of excellent appetite. Appearance is normal, except for slightly protruding eyeballs, and the parents report the child has had difficulty sleeping lately. The nurse should do which of the following? 1. Prepare the family for a neurology consult. 2. Explain the need for an ophthalmology consult. 3. Discuss the plan for thyroid function tests. 4. Explain the plan for an 8-hour fasting blood glucose test.

3 57. 1. The patient exhibits signs of Graves disease, a primary type of hyperthyroidism in children. A neurology consultation is not indicated. 2. Despite the exophthalmos, an eye consultation is not indicated. 3. Diagnostic evaluation for hyperthyroidism is based on thyroid function tests. It is expected in this case that T4 and T3 levels would be elevated, because the thyroid gland is overfunctioning. 4. Fasting blood glucose is used to help evaluate for other endocrine disorders, such as Cushing syndrome and DA.

60. Which information will be most helpful in teaching parents about the primary prevention of foreign body aspiration? 1. Signs and symptoms of foreign body aspiration. 2. Therapeutic management of foreign body aspiration. 3. Most common objects that toddlers aspirate. 4. Risks associated with foreign body aspiration.

3 60. 1, Teaching the parents signs and symptoms of foreign body aspiration is important, but it is a tertiary means of prevention and will not help the parents prevent the aspiration. 2. Teaching the parents the therapeutic management of foreign body aspiration is important, but it is a tertiary means of prevention and will not help the parents prevent the aspiration. 3. Teaching parents the most common objects aspirated by toddlers will help them the most. Parents can avoid having those items in the household or in locations where toddlers may have access to them. 4. Teaching the parents the risks associated with foreign body aspiration is important, but it is a tertiary means of prevention and will not help the parents prevent the aspiration.

62. A teen has just been diagnosed with type I DM. He asks the nurse why he can't take a pill as his cousin does. Which is the nurse's best response? 1. "Once you become an adult, you will be able to take a pill." "Your cousin must be taking the pill for some other reason." 3. "A pill will not treat the type of diabetes you have." 4. "Your doctor will test you to see if you can take the pill."

3 62. 1. Children and adolescents with type 1 DM have to use insulin because they have an absence of insulin from the pancreas. 2. Children and adolescents with type 1 DM have to use insulin because they have an absence of insulin from the pancreas. 3. Children and adolescents with type 1 DM have to use insulin because they have an absence of insulin from the pancreas. 4. Children and adolescents with type 1 DM have to use insulin because they have an absence of insulin from the pancreas.

8. Which combination of signs is commonly associated with glomerulonephritis? 1. Massive proteinuria, hematuria, decreased urinary output, and lethargy. 2. Mild proteinuria, increased urinary output, and lethargy. 3. Mild proteinuria, hematuria, decreased urinary output, and lethargy. 4. Massive proteinuria, decreased urinary output, and hypotension.

3 8. 1. Unlike nephrotic syndrome, protein is lost in mild to moderate amounts. 2. Urinary output is decreased in the child with glomerulonephritis. 3. Mild to moderate proteinuria, hema-turia, decreased urinary output, and lethargy are common findings in glomerulonephritis. 4. Hypertension, not hypotension, is a common finding in glomerulonephritis.

9. The family of a young child has been told the child has DI. What information should the nurse emphasize to the family? 1. One caregiver needs to learn to give the injections of vasopressin. 2. Children should wear MedicAlert tags if they are over 5 years old. 3. Diabetes insipidus is different from diabetes mellitus. 4. Over time, the child may grow out of the need for medication.

3 9. 1. Training two caretakers in the administration of the vasopressin reinforces the importance of the need to give the medication and ensures that medication can be given when the primary caretaker is unable to administer it. For someone on long-term injectable medication, two people need to know how to administer it for the preceding reason. 2. Children should wear MedicAlert tags as soon as they are diagnosed. 3. Explaining that DI is different from DM is crucial to the parents' understanding of the management of the dis-ease. DI is a rare condition that affects the posterior pituitary gland, whereas DM is a more common condition that affects the pancreas. 4. Children with DI do not grow out of their condition and will require close medical management and medications for the rest of their lives.

3. A child had a UTI 3 months ago and was treated with an oral antibiotic. A follow-up urinalysis revealed normal results. The child has had no other problems until this visit when the child is complaining of burning with urination. Which is the most appropriate plan? Select all that apply. 1. Obtain urinalysis and urine culture. 2. Evaluate for renal failure. 3. Admit to the pediatric unit. 4. Send home on an antibiotic if the urinalysis is positive. 5. Schedule a VCUG.

3. 1, 4. 1. Urinalysis and urine culture are routinely used to diagnose UTIs. VCUG is used to determine the extent of urinary tract involvement when a renal ultrasound shows scarring or possible reflux. If the child has a UTI related to bubble baths, constipation, or wiping back to front, a VCUG would not be ordered. 2. There are no data to suggest that renal failure should be evaluated. 3. A UTT is usually treated with oral antibiotics at home and does not routinely require admission to the hospital. 4. If the urinalysis is positive, an antibiotic will be ordered to begin today. 5. If the child has a UTI related to bubble baths, constipation, or wiping back to front, a VCUG would not be ordered.

35. What should the parent of a child with DI be taught about administering desmopressin acetate nasal spray? Select all that apply. 1. The use of the flexible nasal tube. 2. Nasal congestion causes this route to be ineffective. 3. The medication should be administered every 48 hours. 4. The medication should be administered every 8 to 12 hours. 5. Overmedication results in signs of SIADH. 6. Nasal sprays do not always work as well as injections.

35. 1, 2, 4, 5. 1. Administering desmopressin acetate via nasal spray is a means of providing the necessary medication in a steady state if it is given using the flexible nasal tube every 8 to 12 hours. This decreases nasal irritation. 2. If the child becomes ill with rhinorrhea, the desmopressin acetate will need to be administered via the buccal mucosa or rectum or the medication changed to tablets. 3. Administering desmopressin acetate via nasal spray is a means of providing the necessary medication in a steady state if it is given using the flexible nasal tube every 8 to 12 hours. 4. Administering desmopressin acetate via nasal spray is a means of providing the necessary medication in a steady state if it is given using the flexible nasal tube every 8 to 12 hours. 5. Side effects of the desmopressin acetate are those of SIADH. 6. Administering desmopressin acetate via nasal spray is a means of providing the necessary medication in a steady state if it is given using the flexible nasal tube every 8 to 12 hours.

11. Which descriptive terms should be used to describe a school-age child with myxedem-atous skin/eyes/hair changes? 1. The skin is oily and scaly. 2. The skin has pale, thickened patches. 3. The skin is moist, the hair is sparse. 4. The eyes are puffy, the hair is sparse, and the skin is dry.

4 11. 1. Oily skin is not associated with a low serum thyroxine level. It is more consistent with hyperthyroidism. 2. Pale, thickened patches of skin are not associated with hypothyroidism. 3. Moist skin and increased perspiration are characteristic of hyperthyroidism. 4. Myxedema, associated with low serum thyroxine and raised thyrotropin levels, is characteristic of hypothyroid dysfunction and presents with swelling or puffiness of the limbs and face, sparse hair, and very dry skin. These signs may be accompanied by slowness of movements and mental dullness.

14. The nurse is instructing a family on the side effects of oral cortisol. What aspects of administering the medication should the nurse emphasize? 1. Weight gain and dietary management. 2. Bitterness of the taste of the medication. 3. Excitability that results from the medication. 4. Taking the medication with food to decrease gastric irritation.

4 14. 1. Weight gain can and should be controllable with appropriate eating habits and adequate exercise. 2. The patient will quickly figure out that the medication tastes bitter and that the taste needs to be masked. 3. Excitability is not a common side effect. 4. Cortisol should be taken with food to decrease gastric irritation.

16. A parent asks the nurse how to prevent the child from having MCNS again. Which is the nurse's best response? 1. "It is very rare for a child to have a relapse after having fully recovered." 2. "Unfortunately, many children have cycles of relapse, and there is very little that can be done to prevent it." 3. "Your child is much less likely to get sick again if sodium is decreased in the diet." 4. "Try to keep your child away from sick children because relapses have been associated with infectious illnesses."

4 16. 1. It is not unusual for a child to experience relapses. 2. Many children do experience relapses, but exposure to infectious illnesses has been linked to relapses. 3. There is no correlation between the consumption of sodium and minimal change nephrotic syndrome. 4 exposure to infectious illness has been ed to the relapseinimal change nephro syndrome

28. A child is working on a school project on glands and asks the clinic nurse to explain the function of the thymus gland. Which answer would the nurse provide? 1. It produces hormones that help with digestion. 2. It is a gland that disappears by the time a baby is born. 3. A major function is to stimulate the pituitary to act as the master gland. 4. The gland helps with immunity in feral life and early childhood.

4 28. 1. The glands that line the gastrointestinal tract contain cells that produce hormones that control and coordinate secretion of digestive enzymes. 2. The thymus gland is well developed in infancy, attains its greatest size at puberty, and then is changed into fatty tissue. 3. The pituitary gland is the master gland for the body. 4. The thymus acts to provide immunity to the very young body.

36. The nurse evaluates postoperative teaching for repair of testicular torsion as successful when the parent of an adolescent says which the following? 1. "I will encourage him to rest for a few days, but he can return to football practice 2. "I will keep him in bed for 4 days and let him gradually increase his activity 3. "I will seek therapy as he ages because he is now infertile." 4. "I will make sure he does testicular self-examination monthly."

4 36. 1. Lifting and strenuous activity should be avoided for 2 to 4 weeks. 2. The child should not be placed on bedrest and should be encouraged to gradually increase activity while resting as necessary. 3. Most cases of testicular torsion involve only one testis, so most children do not become infertile. 4. The child and family should be taught the importance of testicular self-examination.

39. The nurse evaluates the parents' understanding of the teaching about an inguinal hernia as successful when they say which of the following? 1. "There are no risks associated with waiting to have the hernia reduced; surgery is done for cosmetic reasons." "It is normal to see the bulge in the baby's groin decrease with a bowel movement." "We will wait for surgery until the baby is older because narcotics for pain control will be required for several days." 4. "It is normal for the bulge in the baby's groin to look smaller when the baby is asleep."

4 39. 1. Surgery is usually done at an early age to avoid incarceration, in which the hernia causes impaired circulation to the surrounding tissue. 2. The hernia tends to look larger when the child strains or has a bowel movement because of increased intra-abdominal pressure. 3. The surgery is usually done on an outpatient basis, and narcotics are not usually needed. 4. The hernia often appears smaller when the child is asleep.

39. The nurse is teaching the family about caring for their 7-year-old, who has been diagnosed with type 1 DM. What information should the nurse provide about this condition? 1. Best managed through diet, exercise, and oral medication. 2. Can be prevented by proper nutrition and monitoring blood glucose levels. 3. Characterized mainly by insulin resistance. 4. Characterized mainly by insulin deficiency.

4 39. 1. Type 2 DM is best managed by diet, exer-cise, and oral medication. 2. Proper diet and monitoring blood glucose are important in type 1 DM, but DM is characterized by insulin deficiency. 3. Although insulin resistance can be one of the factors in type 1 DM, it is not the primary factor. 4. Individuals with type 1 DM do not produce insulin. If one does not produce insulin, type 1 DM is the diagnosis.

4. Which should the nurse teach a group of girls and parents about the importance of preventing UTIs? 1. Avoiding constipation has no effect on the occurrence of UTIs. 2. After urinating, always wipe from back to front to prevent fecal contamination. 3. Hygiene is an important preventive measure and can be accomplished with frequent tub baths. 4. Increasing fluids will help prevent and treat UTIs.

4 4. 1. The increased pressure associated with evacuating hardened stool can result in the backflow of urine into the bladder, leading to infection. 2. To prevent infection, a female child should wipe from front to back. 3. Tub baths are not recommended because they may cause irritation of the urethra, leading to infection. 4. Increasing fluids will help flush the bladder of any organisms, encourage urina-tion, and prevent stasis of urine.

41. The nurse caring for a client with type 1 DM is teaching how to self-administer insulin. Which is the proper injection technique? 1. Position the needle with the bevel facing downward before injection. 2. Spread the skin prior to intramuscular injection. 3. Aspirate for blood return prior to injection. 4. Elevate the subcutaneous tissue before injection.

4 41. 1. Correct needle position is with the bevel facing upward. 2. Injection is subcutaneous, so tissue is not spread as it would be for intramuscular injection. 3. Aspiration for blood is not recommended for subcutaneous injections. 4. Skin tissue is elevated to prevent injection into the muscle when giving a subcutaneous injection. Insulin is given only subcutaneously.

41. Which physical findings would be of most concern in an infant with respiratory distress? 1. Tachypnea. 2. Mild retractions. 3. Wheezing. 4. Grunting.

4 41. 1. Tachypnea, an increase in respiratory rate, should be monitored but is a common symptom of respiratory distress. 2. Retractions should be monitored; they can occur with respiratory distress. 3. Wheezing should be monitored and can occur with respiratory distress. 4. Grunting is a sign of impending respiratory failure and is a very concerning physical finding.

The parent of a 3-year-old is shocked to hear the diagnosis of Wilms tumor and says, "How could I have missed a lump this big?" Which is the nurse's best response? 1. "Do not be hard on yourself. It's easy to overlook something that has probably been growing for months when we see our children on a regular basis." 2. "I understand you must be very upset. Your child would have had a better prognosis had you caught it earlier." 3. "It really takes a trained professional to recognize something like this." 4, 4 Dev day blame yourself This mass grows so fast that it was probably not noticabl

4 41. 1. Wilms tumor grows very rapidly and doubles in size in less than 2 weeks. 2. This response places blame on the parent. Wilms tumor has a very good progno-sis, even when first diagnosed at a more advanced stage. 3. This response is condescending and does not acknowledge the parent as the person who knows the child best. 4. The tumor is fast-growing and could very easily not have been evident a few days earlier.

42. The nurse is caring for a child who complains of constant hunger, constant thirst, frequent urination, and recent weight loss without dieting. Which can the nurse expect to be included in care for this child? 1. Limiting daily fluid intake. 2. Weight management consulting. 3. Strict intake and output monitoring. 4. Frequent blood glucose testing.

4 42. 1. Limiting fluids is appropriate for a child presenting with the symptoms of DI, not DM. 2. Weight loss without the other presenting symptoms might be indicative of a need for a weight/nutrition consult. 3. Strict intake and output monitoring is included in the care of a child with DI. 4. Frequent blood glucose testing is included in the care of a child with type 1 DM. The symptoms described in the question are characteristic of a child just prior to the diagnosis of type 1 DM.

The parents of a 7-year-old tell the nurse they do not understand the difference between CKD and AKI. Which is the nurse's best response? 1. "There really is not much difference because the terms are used interchangeably." "Most children experience AKI. It is highly unusual for a child to experience CKD." 3. "CKD tends to occur suddenly and is irreversible." 4. "AKI is often reversible, whereas CKD results in permanent deterioration of kidney function."

4 47. 1. Both disease processes are characterized by the kidney's inability to excrete waste. CKD occurs gradually and is irreversible, whereas AKI occurs suddenly and may be reversible. 2. Children can experience CKD and AKI. 3. CKD is irreversible, and it tends to occur gradually. 4. AKI is often reversible, whereas CKD results in permanent deterioration of kidney function.

47. A 7-year-old is tested for DI. Twenty-four hours after his fluid restriction has begun, the nurse notes that his urine continues to be clear and pale, with a low specific gravity. Which is the most likely reason for this? 1. Twenty-four hours is too early to evaluate effects of fluid restriction. 2. The urine should be concentrated, and it is unlikely the child has DI. 3. The child may have been sneaking fluids and needs closer observation. 4. In DI, fluid restriction does not cause urine concentration.

4 47. 1. Within 24 hours of fluid restriction, the urine becomes concentrated in a healthy child. 2. Because the urine is not concentrated, the child is likely to have DI. 3. Children should be carefully observed to make sure they are not sneaking fluids; however, the assumption that this child has been doing so should not be made. 4. Children with DI cannot concentrate urine.

48. The nurse has completed discharge teaching for the family of a 10-year-old diagnosed with DI. Which statement best demonstrates the family's correct understanding of DI? 1. "The disease was probably brought on by a bad diet and little exercise." 2. "Diabetes seems to run in my family, and that may be why my child has it." 3. "My child will need to check blood sugar several times a day." 4. "My child will have to use the bathroom more often than other children."

4 48. 1. The primary causes of DI are idiopathic, organic, and brain trauma, unlike the causes for type 2 DM, for which diet and exercise are major factors. 2. DI does not have a hereditary factor. 3. Blood sugar monitoring is important with DM, not DI. 4. Despite the use of vasopressin to treat the symptoms of DI, breakthrough urination is likely.

48. What should be the nurse's first action with a child who has a high fever, dysphagia, drooling, tachycardia, and tachypnea? 1. Immediate IV placement. 2. Immediate respiratory treatment. 3. Thorough physical assessment. 4. Lateral neck radiographs.

4 48. 1. This child is exhibiting signs and symptoms of epiglottitis and should be kept as comfortable as possible. Agitating the child may cause increased airway swelling and may lead to complete obstruction. 2. Respiratory treatments often frighten children. This child is exhibiting signs and symptoms of epiglottitis and should be kept as comfortable as possible. Agitating the child may cause increased airway swelling and may lead to complete obstruction. 3. This child is exhibiting signs and symptoms of epiglottitis and should be kept as comfortable as possible. Agitating the child may cause increased airway swelling and may lead to complete obstruction. The child should be allowed to remain on the parent's lap and be kept as comfortable as possible until a lateral neck film is obtained. 4. This child is exhibiting signs and symptoms of epiglottitis and should be kept as comfortable as possible. The child should be allowed to remain in the parent's lap until a lateral neck film is obtained for a definitive diagnosis.

5. A 12-year-old comes to the clinic with a diagnosis of Graves disease. What information should the nurse discuss with the child? 1. Suggest weight loss. 2. Encourage attending school. 3. Emphasize that the disease will go into remission. 4. Encourage the child to take responsibility for daily medications.

4 5. 1. Children with Graves disease have voracious appetites and lose weight. 2. Attending school and continuing typical activities is beneficial in terms of long-term management. Gym class and after-school sports should be restricted until the child is euthyroid. 3. Graves disease may go into remission after 2 or 3 years; for some children, however, it does not. 4. Because the child is 12 years old, encouraging responsibility for health care is important. The child still needs family involvement and ongoing supervision but should not be completely dependent on family for care.

51. The parent of a child with croup tells the nurse that her other child just had croup and it cleared up in a couple of days without intervention. She asks the nurse why this child is exhibiting worse symptoms and needs to be hospitalized. Which is the nurse's best response? 1. "Some children react differently to viruses. It is best to treat each child as an individual." 2. "Younger children have wider airways that make it easier for bacteria to enter and colonize." 3. "Younger children have short and wide eustachian tubes, making them more susceptible to respiratory infections." 4. "Children younger than 3 years usually exhibit worse symptoms because their Immune systems are not as developed."

4 51. 1. All children should be treated as individuals when they are being treated for a particular illness. However, most children exhibit similar symptoms when they have the same diagnosis. Younger children have worse symptoms than older children because their immune systems are less developed. 2. Children have airways that are shorter and narrower than those of adults. As children age, their airways begin to grow in length and diameter. 3. Children are more prone to ear infections because they have eustachian tubes that are short and wide and lie in a horizontal plane. 4. Younger children have less developed immune systems and usually exhibit worse symptoms than older children.

53. Which intervention is best to teach the mother of a child diagnosed wich a URI and a dry, hacking cough that prevents him from sleeping? 1. Give cough suppressants at night. 2. Give an expectorant every 4 hours. 3. Give cold and flu medication every 8 hours. 4. Give ½ teaspoon of honey four to five times per day.

4 53. 1. Cough suppressants are not recommended for children. Coughing is a protective mechanism, so do not try to stop it. 2. Expectorants are not recommended for children younger than 6 years of age. There is no research showing that they are effective. 3. Cold and flu medications are not indicated for children younger than 6 years of age, as there is no indication they are effective. 4. Warm fluids, humidification, and honey are the best treatments for a URI. But honey is not used in children under 1 year of age because of the association with botulism.

53. Which is the reason a student takes metformin (Glucophage) three times a day? 1. Type 1 DM. 2. DI. 3. Inflammatory bowel disease. 4. Type 2 DM.

4 53. 1. Type 1 DM is managed with insulin, not oral agents. 2. DDAVP, not metformin, is used to treat diabetes insipidus. 3. Methylprednisolone (Medrol) can be used to treat inflammatory bowel disease. 4. Metformin is commonly used to manage type 2 DM.

57. A child with severe cerebral palsy is admitted to the hospital with aspiration pneumo-nia. What is the most beneficial educational information that the nurse can provide to the parents? 1. The signs and symptoms of aspiration pneumonia. 2. The treatment plan for aspiration pneumonia. 3. The risks associated with recurrent aspiration pneumonia. 4. The prevention of aspiration pneumonia.

4 57. 1. The nurse should teach the parents about the signs and symptoms of aspiration pneu-monia. The most valuable information relates to preventing aspiration pneumonia from occurring in the future. 2. The nurse should instruct the parents on the treatment plan of aspiration pneu-monia, but that is not the most beneficial piece of information the nurse can provide. The most valuable information relates to preventing aspiration pneumonia from occurring in the future. 3. The nurse should instruct the parents on the risks associated with recurrent aspiration pneumonia, but that is not the most beneficial piece of information the nurse can provide. The most valuable information relates to preventing aspiration pneumonia from occurring in the future. 4. The most valuable information the nurse can give the parents is how to prevent aspiration pneumonia from occurring in the future.

6. Which is the best way to obtain a urine sample in an 8-month-old being evaluated for a UTI? 1. Carefully cleanse the perineum from front to back and apply a self-adhesive urine collection bag to the perineum. 2. Insert an indwelling Foley catheter, obtain the sample, and wait for results. 3. Place a sterile cotton ball in the diaper and immediately obtain the sample with a syringe after the first void. 4. Using a straight catheter, obtain the sample, and immediately remove the catheter without waiting for the results of the urine sample.

4 6. 1. A sample obtained from a urine bag would contain microorganisms from the skin, causing contamination of the sample. 2. There is no need to leave the catheter in because it serves as a portal for infection. 3. The cotton ball would not remain sterile and would therefore contaminate the urine sample. 4. An in-and-out catheterization is the best way to obtain a urine culture in a child who is not yet toilet-trained.

45. What information should the nurse provide the parent of a child diagnosed with nasopharyngitis? 1. Complete the entire prescription of antibiotics. 2. Avoid sending the child to day care. 3. Use comfort measures for the child. 4. Restrict the child to clear liquids for 24 hours.

45. 3 1. Nasopharyngitis is a viral illness and does not require antibiotic therapy. 2. Children who attend day care are more prone to catching viral illnesses, but it is not the nurse's place to tell the parents not to send their child to day care. Often, families do not have a choice about using day care 3. Nursing care for nasopharyngitis is primarily supportive. Keeping the child comfortable during the course of the illness is all the parents can do. Nasal congestion can be relieved using normal saline drops and bulb suction. Acetaminophen (Tylenol) can also be given for discomfort or a mild fever. 4. There is no reason to restrict the child to clear liquids. Many children have a decreased appetite during a respiratory illness, so the most important thing is to keep them hydrated.

46. Which assessment is of greatest concern in a 15-month-old? 1. The child is lying down and has moderate retractions, low-grade fever, and nasal congestion. 2. The child is in the tripod position and has diminished breath sounds and a muffled cough. 3. The child is sitting up and has coarse breath sounds, coughing, and fussiness. 4. The child is restless and crying, has bilateral wheezes, and is feeding poorly..

46. 2 1. Retractions indicate some degree of respiratory distress, but more information needs to be obtained. Other common symptoms of a respiratory illness, including low-grade fever and nasal congestion, are not overly concerning. 2. When children are sitting in the tripod position, they are having difficulty breathing. The child is sitting and leaning forward in order to breathe more easily. Diminished breath sounds are indicative of a worsening condi-tion. A muffled cough indicates that the child has some subglottic edema. This child has several signs and symptoms of a worsening respiratory condition. 3. Coarse breath sounds, cough, and fussiness are common signs and symptoms of a respiratory illness. 4. Restlessness, wheezes, poor feeding, and crying are signs and symptoms of a respiratory illness.

49. The nurse is interviewing the parent of a 9-year-old girl. The parent expresses concern because the daughter already has pubic hair and is starting to develop breasts. Which statements would be most appropriate? Select all that apply. 1. "Your daughter should get her period in approximately 6 months." 2. "Your daughter is developing early and should be evaluated for precocious puberty." 3. "Your daughter is experiencing body changes that are appropriate for her age." 4. "Your daughter will need further testing to determine the underlying cause." 5. "Your daughter will need sexual counseling in the next year or two."

49. 3, 5. 1. Menarche usually occurs within 2 years after the start of breast development. 2. Age 9 is the average age of thelarche (breast bud development), which is followed shortly by the appearance of pubic hair. 3. The changes described in the question are normal for a healthy 9-year-old female. 4. No further testing is required because this child's development is normal. 5. Sexual counseling does need to be provided in the next couple of years.

5. Which child does not need a urinalysis to evaluate for a UTI? 1. A 4-month-old female presenting with a 2-day history of fussiness and poor appetite; current vital signs include axillary temperature of 100.8°F (38.2°C) and heart rate of 120 beats per minute. 2. A 4-year-old female who states, "It hurts when I pee"; she has been urinating every 30 minutes; vital signs are within normal range. 3. An 8-year-old male presenting with a finger laceration; mother states he had surgical re-implantation of his ureters 2 years ago. 4. A 12-year-old female complaining of pain to her lower right back; she denies any burning or frequency at this time; oral temperature of 101.5°F (38.6°C).

5. 3 1. Fussiness and lack of appetite can indicate a UTI. Signs of infection, such as fever and increased heart rate, should be evaluated to determine whether an infection exists. 2. Frequency and urgency are classic signs of a UTI. 3. Although this child has had a history of urinary infections, the child is currently not displaying any signs and therefore does not need a urinalysis at this time. 4. Pain to the lower right back can indicate infection of the upper urinary tract. Although the child currently denies any burning or frequency, the child currently has a fever coupled with flank pain, which needs evaluation.

50. A mother is crying and tells the nurse that she should have brought her son in yesterday when he said his throat was sore. Which is the nurse's best response to this parent whose child is diagnosed with epiglottitis and is in severe distress and in need of intubation? 1. "Children this age rarely get epiglottis; you should not blame yourself." 2. "It is always better to have your child evaluated at the first sign of illness rather than wait until symptoms worsen." 3. "Epiglottitis is slowly progressive, so early intervention may have decreased the extent of your son's symptoms." 4. "Epiglottis is rapidly progressive; you could not have predicted his symptoms would worsen so quickly."

50. 4 1. Epiglottitis is most common in children from 2 to 5 years of age. The onset is very rapid. Telling parents not to blame themselves is not effective. Parents tend to blame themselves for their child's illnesses even though they are not responsible. 2. The nurse should not tell the parent to seek medical attention for any and all signs of illness. 3. Epiglottitis is rapidly progressive and cannot be predicted. 4. Epiglottitis is rapidly progressive and cannot be predicted.

58. A 12-year-old with hyperthyroidism is being treated with standard antithyroid drug therapy. A parent calls the office stating that the child has a sore throat and fever. Which is the nurse's best response? 1. "Bring your child to the office or emergency department immediately." "Slight fever and sore throat are normal side effects of the medication." 3. "Give your child the appropriate dose of ibuprofen and call back if symptoms worsen." 4. "Give your child at least 8 oz of clear fluids and call back if symptoms worsen."

58. 1. A complication of antithyroid drug therapy is leukopenia. Fever and sore throat, therefore, need to be evaluated immediately. This is an essential component of discharge teaching for patients with Graves disease. Common side effects of antithyroid drugs include rash, nausea, vomiting, headache, drowsiness, but not sore throat. This is a tempting choice for the test taker, because fever and sore throat appear to be fairly benign symptoms. Because the question includes information regarding Graves disease and medication therapy, however, the test taker should eliminate this answer. 4. It is most likely that medication, not fluid Status, contributes to the child's symptoms. Pairing Token: 25bf-779a

62. A chloride level greater than. __ of CF. is a positive diagnostic indicator

60 mEq/L. The definitive diagnosis of CF is made when a child has a sweat chloride level >60 mEg/L. A normal chloride level is <40 mEq/L.

62. Which is true of a Wilms tumor? Select all that apply. 1. It is also referred to as neuroblastoma. 2. It can occur at any age but is seen most often between the ages of 2 and 5 years. 3. It can occur on its own or can be associated with many congenital anomalies. 4. It is a slow-growing tumor. 5. It is associated with a poor prognosis.

62. 2, 3. 1. It is referred to as a nephroblastoma, not a neuroblastoma. 2. It can occur at any age but is seen most often between the ages of 2 and 5 years. 3. It can occur on its own or can be associated with many congenital anomalies. 4. It is a tumor that grows very quickly. 5. It is associated with a very good prognosis.

63. Which of the following signs/symptoms would the nurse teach the family of a child newly diagnosed with type I DM to recognize as hypoglycemia? Select all that apply. 1. Pallor. 2. Confusion. 3. Tachycardia. 4. Sweating. 5. Acetone breath.

63. 1, 2, 3, 4. 1. Hypoglycemia is caused by taking too much insulin, skipping a meal or not eating as much as the child should, and strenuous exercise without adequate dietary replacement. Symptoms of hypoglycemia are pallor, confusion, tachycardia, and sweating. 2. Hypoglycemia is caused by taking too much insulin, skipping a meal or not eating as much as the child should, and strenuous exercise without adequate dietary replacement. Symptoms of hypoglycemia are pallor, confusion, tachycardia, and sweating. 3. Hypoglycemia is caused by taking too much insulin, skipping a meal or not eating as much as the child should, and strenuous exercise without adequate dietary replacement. Symptoms of hypoglycemia are pallor, confusion, tachycardia, and sweating. 4. Hypoglycemia is caused by taking too much insulin, skipping a meal or not eating as much as the child should, and strenuous exercise without adequate dietary replacement. Symptoms of hypoglycemia are pallor, confusion, tachycardia, and sweating. 5. Acetone breath is detected when a child is hyperglycemic. Confusion, deep and slow breathing, thirst, weakness, tiredness, and headache are also present.

63. A 4-week-old infant presents to the clinic with a temperature of 101.6°F (38.5°C), fussiness, decreased appetite, and normal physical examination. Which is the next management step? Select all that apply. 1. Perform a urinary catheterization. 2. Start IV fluids. 3. Draw serum electrolytes. 4. Draw a blood culture. 5. Administer acetaminophen (Tylenol).

63. 1, 5. 1. An infant who presents with a fever and no other symptoms would be worked up for an infection, including a urinary catheterization, complete blood count, and, if needed, a lumbar puncture. 2. IV Auids would not be started yet unless the infant is moderately dehydrated. Many clinics do not start IV Auids but would send the infant to the emergency department if fluids were needed immediately.

64. Which of the following characteristics would a nurse expect in a 2-month-old just diagnosed with hypothyroidism? Select all that apply. 1. Protruding tongue. 2. Hypotonia. 3. Open fontanels. 4. Lethargy. 5. Tachycardia.

64. 1, 2. 1. Common findings in an infant with hypothyroidism include protruding tongue, hypotonia, and bradycardia. 2. Common findings in an infant with hypothyroidism include protruding tongue, hypotonia, and bradycardia. 3. Posterior fontanel closes at 6 to 8 weeks of age. Anterior fontanel closes at 12 to 18 months. 4. Symptoms of hyperthyroidism include tachycardia, weight loss, fatigue, diarrhea, and tremors. 5. Symptoms of hyperthyroidism include tachycardia, weight loss, fatigue, diarrhea, and tremors.

64. Which of the following laboratory results would be of concern? Select all that apply. 1. Urinalysis with 1+ protein. 2. Urinalysis with positive nitrites. 3. Blood urea nitrogen of 15 mg/dL. 4. Serum creatinine of 0.7 umol/L 5. Colony count of 50,000 of a bacterium.

64. 2, 5. 1. A urinalysis with positive protein of 1+ or 2+ can be normal if obtained at the end of the day. It's called "orthostatic proteinuria." 2. Positive nitrites on a urinalysis is almost always an indication of a UTI. 3. A normal blood urea nitrogen is 8 to 20 mg/dL. 4. A normal serum creatinine is 0.6 to 1.2 umol/L 5. A colony count on a urine culture of 50,000 or more is considered positive, and treatment needs to start.

I21 A 10-year-old with severe factor VIII deficiency falls, injures an elbow, and is brought to the ED. The nurse should prepare which of the following? 1. An IM injection of tactor VIll. 2. An IV infusion of tactor VIII 3. An Injection of desmopressin 4 An IV infusion of platelets

2 21. 1. Factor VIII is not given intramuscularly. 2. The child is treated with an IV infusion of factor VIII to replace the missing factor and help stop the bleeding. 3. Desmopressin is given to stimulate factor VIlI production, and it is given intravenously.

21. Which child with asthma should the nurse see first? 1. A 12-month-old who has a mild cry, is pale in color, has diminished breath sounds, and has an oxygen saturation of 93%. 2. A 5-year-old who is speaking in complete sentences, is pink in color, is wheezing bilaterally, and has an oxygen saturation of 93% 3. A 9-year-old who is quiet, is pale in color, and is wheezing bilaterally with an oxygen saturation of 92%. 4. A 16-year-old who is speaking in short sentences, is wheezing, is sitting upright, and has an oxygen saturation of 93%.

1 21. 1. This child is exhibiting signs of severe asthma. This child should be seen first. The child no longer has wheezes and now has diminished breath sounds. 2. This child is exhibiting symptoms of mild asthma and should not be seen before the other children. 3. This child is exhibiting signs of moderate asthma and should be watched but is not the patient of highest priority. 4. This child is exhibiting signs of moderate asthma and is not the patient of highest priority.

25. Which of the following measures should be implemented for a child with von Wille. brand disease who has a nosebleed: 1. Apply pressure to the nose for at least Ominutes. 2. Have the child lie supine and quiet. 3. Avoid packing of the nostrils. 4. Encourage the child to swallow frequentlv.

1 25. 1. Applying pressure to the nose may stop the bleeding. In von Willebrand disease, there is an increased tendency to bleed from mucous membranes, leading to nosebleeds commonly from the anterior part of the nasal septum. 2. The child should sit up and lean forward to avoid aspiration of blood. 3. Packing the nose with cotton may stop bleeding, but be careful when the cotton is removed because it may dislodge the clot. 4. Swallowing will cause the child to swallow the blood, which then can cause vomiting.

26. Which would be appropriate nursing care management of a child with the diagnosis of mononucleosis? 1. Only family visitors. 2. Bedrest. 3. Clear liquids. 4. Limited daily fluid intake.

1 26. 1. Children with mononucleosis are more susceptible to secondary infections. Therefore, they should be limited to visitors within the family, especially during the acute phase of illness. 2. Children with mononucleosis do not need to be on bedrest. Children usually self-limit their activities. 3. Children with mononucleosis do not need a restricted diet. Often, they are very tired and are not interested in eating. The nurse and family must ensure that the children are taking in adequate nutrition. 4. Children with mononucleosis usually have decreased appetite, but it is essential that they remain hydrated. There is no reason to restrict fluid intake.

28. Who is at the highest priority to receive the fu vaccine? 1. A healthy 8-month-old who attends day care. 2. A 3-year-old who is undergoing chemotherapy. 3. A healthy 7-year-old who attends public school. 4. An 18-year-old who is living in a college dormitory.

1 28. 1. Children between the ages of 6 and 23 months are at the highest risk for having complications as a result of the fu. Their immune systems are not as developed, so they are at a higher risk for influenza-related hospitalizations. 2. The flu vaccine should not be given to anyone who is immunocompromised. 3. The flu vaccine is recommended for all ages, but the 7-year-old is not the highest priority. A child this age will likely recover without any complications. 4. The flu vaccine is recommended for all ages, but the 18-year-old is not the highest priority. A person this age will likely recover without any complications.

The parent of a child with in Auenza asks the nurse when the child is most infectious. Which is the nurse's best response? 1. "Twenty-four hours before and after the onset of symptoms." 2. "Twenty -four hours after the onset of symptoms." 3. "One week after the onset of symptoms." 4. "One week before the onset of symptoms."

1 29. 1. Influenza is most contagious 24 hours before and 24 hours after onset of symptoms. 2. Influenza is most contagious 24 hours before and 24 hours after onset of symptoms. 3. Influenza is most contagious 24 hours before and 24 hours after onset of symptoms. 4. Influenza is most contagious 24 hours before and 24 hours after onset of symptoms.

32. Which is the nurse's best response to a parent who asks what can be done at home to help an infant with URI symptoms and a fever get better? 1. "Give your child small amounts of fluid every hour to prevent dehydration." 2. "Give your child Robitussin at night to reduce his cough and help him sleep." 3. "Give your child a baby aspirin every 4 to 6 hours to help reduce the fever." 4. "Give your child an over-the-counter cold medicine at night."

1 32. 1. It is essential that parents ensure their children remain hydrated during a URI. The best way to accomplish this is by giving small amounts of fluid frequently. 2. Over-the-counter cough and cold medicine is not recommended for any child younger than 6 years. 3. Aspirin is not given to children to treat a viral infection because of the risk of developing Reye syndrome. 4. Over-the-counter cough and cold medicine is not recommended for any child younger than 6 years.

39. Which statement indicates the parent needs further teaching on how to prevent his other children from contracting RSV? 1. "I should make sure that both my children receive palivizumab (Synagis) injections for the remainder of this year." 2. "I should be sure to keep my infected child away from his brother until he has recovered 3. "I should insist that all people who come in contact with my children thoroughly wash their hands before playing with them." 4. "I should insist that anyone with a respiratory illness avoid contact with my children until well."

1 39. 1. Palivizumab will not help the child who has already contracted the illness. Palivizumab is an immunization and a method of primary prevention. 2. RSV is spread through direct contact with respiratory secretions, so it is a good idea to keep the ill child away from the healthy one. 3. RSV is spread through direct contact with respiratory secretions, so it is a good idea to have all persons coming in contact with the child wash their hands. 4. RSV is spread through direct contact with respiratory secretions, so it is a good idea to have ill persons avoid any contact with the children until they are well.

64 According to the American Academy of Pediatrics, at which age should an infant have their first blood drawn to test for lead level? 1. 12 months. 2. 15 months. 3.8 months. 4. 20 months.

1 64. 1. According to the American Academy of Pediatrics, the appropriate ages for testing for blood lead levels is at the 9- or 12-month well-child visit. 2. According to the American Academy of Pediatrics, the appropriate ages for testing for blood lead levels is at the 9- or 12-month well-child visit. 3. According to the American Academy of Pediatrics, the appropriate ages for testing for blood lead levels is at the 9- or 12-month well-child visit. 4. According to the American Academy of Pediatrics, the appropriate ages for testing for blood lead levels is at the 9- or 12-month well-child visit.

19. The nurse is caring for a child with sickle cell disease who is scheduled to have an exchange transfusion. What information should the nurse teach the family? 1. The procedure is done to prevent further sickling during a vaso-occlusive crisis 2. The procedure reduces side effects from blood transtusions. 3. The procedure is a routine treatment for sickle cell crisis. 4. Once the child's spleen is removed, it is not necessary to do exchange transtusions.

1 19. 1. Exchange transfusion reduces the number of circulating sickle cells and slows down the cycle of hypoxia, thrombosis, and tissue ischemia. 2. Exchange transfusion does not decrease risk of a transfusion reaction. Every time a transfusion is done, the child continues to be at risk for a reaction. 3. This is not a routine procedure and is performed only when the number of sickle cells is elevated and the child is at high risk for thrombosis. 4. After a splenectomy, transfusions still need to be done depending on the client's hemo- globin level.

A 3-year-old admitted to the hospital with croup has the following vital signs: heart rate 90, respiratory rate 44, blood pressure 100/52, and temperature 98.8°F (37.1°C). The parents ask the nurse if these vital signs are normal. The nurse's best response is: 1. "Your son's blood pressure is elevated, but the other vital signs are within the normal range." 2. "Your son's temperature is elevated, but the other vital signs are within the normal range." 3. "Your son's respiratory rate is elevated, but the other vital signs are within the normal range." 4. "Your son's heart rate is elevated, but the other vital signs are within the normal range."

1. A normal systolic blood pressure for a child from 3 to 6 years is 78 to 111. A normal diastolic blood pressure for a child from 3 to 6 years is 42 to 70. 2. A normal temperature is 96.6°F to 100°F (35.8°C to 37.7°C). ANSWER 3 A normal respiratory rate for a child from 3 to 6 years is 20 to 30 breaths per minute. 4. A normal heart rate for a child from 3 to 6 years is 75 to 120.

Which technique should the nurse suggest to the mother of an 8-year-old who does not want to complete her chores? 1. Grounding. 2. Time-out. 3. Reward system 4. Spanking.

1. Grounding is a technique that generally works well with adolescents. 2. Time-out is a technique that is primarily used for toddlers and preschool children. ANS School-age children usually respond very well to a reward system and often enjoy the rewards so much that they will continue chores without continual reminders. 4. Spanking is a suggestion that should never be given to families.

Which action is a developmentally appropriate method for eliciting a 4-year-old's cooperation in obtaining the blood pressure? 1. Have the child's parents help put on the blood pressure cuff. 2. Tell the child that if he sits still, the blood pressure machine will go quickly. 3. Ask the child if he feels a squeezing of his arm, 4. Tell the child that measuring the blood pressure will not hurt.

1. Preschool children like to do things for themselves and will not likely behave any better for the parents than for the nurse. 2. The nurse should not promise the child that the procedure will go quickly. The nurse needs to develop a trusting relationship with the child; therefore, only promises that can be kept should be made. ANSWER 3 Preschool children enjoy games, and it is a good way to elicit their assistance and cooperation during a procedure. 4. The nurse should not promise the child that the procedure will not hurt. Each child's perception of pain is individual. The nurse needs to develop a trusting relationship with the child; therefore, only promises that can be kept should be made.

17. What is the most important piece of information that the nurse must ask the parent of a child in status asthmaticus? 1. "What time did your child eat last?" 2. "Has your child been exposed to any of the usual asthma triggers?" 3. "When was your child last admitted to the hospital for asthma?" 4. "When was your child's last dose of medication?"

17. 4 1. The nurse needs to know when the child ate last in the event that the child may need to be intubated for severe respiratory distress, but it is not the most vital piece of information to best treat the child for the current state of distress. 2. The nurse needs to know if the child was exposed to anything that usually triggers the asthma, but that is not the most important information for treating the child's immediate need. 3. Knowing when the child was admitted last will give the nurse an idea of the severity of the child's asthma, but that is not the most important information for treating the child's immediate need. 4. The nurse needs to know what medication the child had last and when the child took it in order to know how to begin treatment for the current asthmatic condition.

18. Which of the following is the most effective treatment for pain in a child with sickle cell crisis? Select all that apply. 1. Meperidine (Demerol). 2. Aspirin. 3. Morphine. 4. Behavioral techniques. 5. Acetaminophen (Ivlenol) with codeine.

18. 3, 4, 5. 1. Meperidine (Demerol) should not be used because it may potentiate seizures. 2. Aspirin should not be used in children because of the risk for Reye syndrome. 3. Morphine is the drug of choice for a child with sickle cell crises. Usually the child is started on oral doses of acetaminophen (Trlenol) with codeine. When that is not sufficient to alleviate pain, stronger narcotics are prescribed, such as morphine. Ketorolac (Toradol) may be indicated for short-term use for moderate-severe pain. 4. Behavioral techniques such as positive self-talk, relaxation, distraction, and guided imagery are helpful when pain is occurring. 5. Usually the child is started on oral doses of acetaminophen (Tylenol) with codeine when pain is described as mild to moderate.

19. Which statement by the parent of a child using an albuterol inhaler leads the nurse to believe that further education is needed on how to administer the medication? 1. "I should administer two quick puffs of the albuterol inhaler using a spacer. 2. "I should always use a spacer when administering the albuterol inhaler." 3. "I should be sure that my child is in an upright position when administering the inhaler. 4. "I should always shake the inhaler before administering a dose."

19. 1, 1 The parent of an asthmatic child should always give one puff at a time and wait 1 minute before administering the second puff. 2. A spacer is recommended when administering medications by MDI to children. 3. The child should be in an upright position when medications are administered by MDI. 4. The inhaler should always be shaken before administering a dose of the medication.

13. A school-age child has been diagnosed with strep throat. The parent asks the nurse when the child can return to school. Which is the nurse's best response? 1. "Forty-eight hours after the first documented normal temperature." 2. "Twenty-four hours after the first dose of antibiotics." 3. "Forty-eight hours after the first dose of antibiotics." 4. "Twenty-four hours after the first documented normal temperature."

2 13. 1. School systems require that children remain home for 24 hours after having a documented fever. However, in this ques-tion, the child has been diagnosed with strep throat. Even if the child is fever-free, the child must have completed a 24-hour course of antibiotics before returning to school. Children with strep throat are no longer contagious 24 hours after initiation of antibiotic therapy. 2. Children with strep throat are no longer contagious 24 hours after initiation of antibiotic therapy. 3. Children with strep throat are no longer contagious 24 hours after initiation of antibiotic therapy. 4. School systems require that children remain home for 24 hours after having a documented fever. However, in this ques-tion, the child has been diagnosed with strep throat. Even if the child is fever-free, the child must have completed a 24-hour course of antibiotics before returning to school. Children with strep throat are no longer contagious 24 hours after initiation of antibiotic therapy.

17. The nurse is caring for a child with sickle cell disease who is scheduled to have a sple- nectomy. What information should the nurse explain to the parents regarding the reason for a splenomegaly. 1. To decrease potential for infection. 2. To prevent splenic sequestration. 3 To prevent sickling of red blood cells. 4. 1o prevent sickle cell crisis.

2 17. 1. The cells involved with sickle cell disease are abnormal red blood cells, which do not decrease infections. 2. Splenic sequestration is a life-threatening situation in children with sickle cell disease. Once a child is considered to be at high risk of splenic sequestration or has had this in the past, the spleen will be removed. 3. Removal of the spleen will not prevent sickling, because it will not change the disease condition. 4. The child will still have sickle cell disease and can still have sickle cell crises.

23. The parent of a child with hemophilia is asking the nurse what caused the hemophilia. Which iS the nurses best response? 1. It is an X-linked dominant disorder. 2. It is an X-linked recessive disorder. 3. It is an autosomal dominant disorder. 4. It is an autosomal recessive disorder.

2 1Hemophilia is transmitted as an X-linked recessive disorder. About 60% of children have a family history of hemophilia. The usual transmission is by a female with the trait and an unaffected male. 2. Hemophilia is transmitted as an X-linked recessive disorder. About 60% of children have a family history of hemophilia. The usual transmission is by a female with the traitsassonat- fected male. 3. Hemophilia is transmitted as an X-linked recessive disorder. About 60% of children have a family history of hemophilia. The usual transmission is by a female with the trait and unaffected male. 4.Hemophilia is transmitted as an X-linked recessive disorder. About 60% of children have a family history of hemophilia. The sual transmission is by a female with the

30. A 6-week-old is admitted to the hospital with influenza. The child is crying, and the father tells the nurse that his son is hungry. 'The nurse explains that the baby is not to have anything by mouth. The parent does not understand why the child cannot eat. Which is the nurse's best response to the parent? 1. "We are giving your child IV fluids, so there is no need for anything by mouth." 2. "The shorter and narrower airway of infants increases their chances of aspiration, so your child should not have anything to eat now." 3. "When your child eats, he burns too many calories; we want to conserve the child's energy." 4. "Your child has too much nasal congestion; if we feed the child by mouth, the distress will likely increase."

2 30. 1. The child is receiving V fluids, so he is being hydrated. However, this response does not explain to the father why his son cannot eat. 2. Infants are at higher risk of aspiration because their airways are shorter and narrower than those of adults. An infant with influenza has increased nasal secretions and coughs up mucus. With excess secretions, the infant is at an even higher risk of aspiration. 3. Eating burns calories, but if the baby is upset and crying, he is also expending energy. Therefore, this is not the best answer choice. 4. If the child has nasal congestion, that may make it difficult for him to feed.

5. The parent of a child with CF is excited about the possibility of the child receiving a double lung transplant. What should the parent understand? 1. The transplant will cure the child of CF and allow the child to lead a long and healthy life. 2. The transplant will not cure the child of CF but will allow the child to have a longer life. 3. The transplant will help to reverse the multisystem damage that has been caused by CF. 4. The transplant will be the child's only chance at surviving long enough to graduate from college.

2 5. 1. A lung transplant does not cure CF, but the transplanted lungs do not contain the CF genes. Although the new lungs do not contain CF, the sinuses, pancreas, intestines, sweat glands, and reproductive tract do. The new lungs are more susceptible to infection because of the immunosuppressive therapy that must be given post-transplant. Immunosuppressive drugs make it difficult for the body to fight infection, which can lead to lung damage. 2. A lung transplant does not cure CF, but it does offer the client an opportunity to live a longer life. The concerns are that, after the lung transplant, the child is at risk for rejection of the new organ and for development of secondary infections because of the immunosuppressive therapy. 3. The lung transplant does not reverse the damage that has been done to the child's other organs, but it does offer a chance of a longer life. 4. The average life span of a client with CF has risen over the years with the daily regimens of CPT, exercise, medications, and high-calorie, high-protein diets.

10. A school-age child has been diagnosed with nasopharyngitis. The parent is concerned because the child has had little or no appetite for the last 24 hours. Which is the nurse's best response? 1. "Do not be concerned; it is common for children to have a decreased appetite during a respiratory illness." 2. "Be sure your child is taking an adequate amount of fluids. The appetite should return soon." 3. "Try offering the child some favorite foods. Maybe that will improve the appetite." 4. "You need to force your child to eat whatever you can; adequate nutrition is essential."

2 10. 1. Children commonly have a decreased appetite when they have a respiratory ill-ness. However, the nurse needs to instruct the parent to offer fluids to ensure the child stays hydrated. 2. Children commonly have a decreased appetite when they have a respiratory illness. The nurse is appropriately instructing the parent that the child will be fine by taking in an adequate amount of fluid to keep the child hydrated. 3. The child may want to eat some favorite foods; however, the child will be fine if an adequate amount of fluid is maintained. 4. The parent should not force the child to eat; the child's appetite should return in a couple of days.

18. Which is the nurse's best response to parents who ask what impact asthma will have on the child's future in sports? 1. "As long as your child takes prescribed asthma medication, the child will be fine." 2. "The earlier a child is diagnosed with asthma, the more significant the symptoms." 3. "The earlier a child is diagnosed with asthma, the better the chance the child has of growing out of the disease " "Your child should avoid playing contact sports and sports that require a lot of running."

2 18. 1. It is essential that the child take all of the scheduled asthma medications, but there is no guarantee the child will be fine and be able to play all sports. 2. When a child is diagnosed with asthma at an early age, the child is more likely to have significant symptoms on aging. 3. Children diagnosed at an early age usually exhibit worse symptoms than those diagnosed later in life. 4. Children with asthma are encouraged to participate in sports.

30 Which test provides a definitive diagnosis of aplastic anemia? 1. Complete blood count with differential. 2. Bone marrow aspiration. 3. Serum IgG levels. 4. Basic metabolic panel.

2 30. 1. A complete blood count would show pan-cytopenia, which would lead the healthcare provider to look for the cause of the blood abnormality. 2. Definitive diagnosis is determined from bone marrow aspiration, which demonstrates the conversion of red bone marrow to yellow, fatty marrow. 3. Serum [gG levels do not diagnose aplastic anemia, which does not seem to have an immune cause. 4. A basic metabolic panel tests for metabolic disorders.

20. The mother of a child 2 years 6 months has arranged a play date with the neighbor and her child 2 years 9 months. During the play date the two mothers should expect that the children will do which of the following? 1. Share and trade their toys while playing. 2. Play with one another with little or no conflict. 3. Play alongside one another but not actively with one another. 4. Only play with one or two items, ignoring most of the other toys.

3. Play alongside one another but not actively with one another. Toddlers engage in parallel play. They often play alongside another child, but they rarely engage in activities with the other child. Toddlers do not share their possessions well. One of their favorite words is "mine."

Which of the following should the nurse expect to administer to a child with ITP and a platelet count of 5000/mm'? Select all that apply. 1. Platelets. 2. Intravenous immunoglobulin. 3. Packed red blood cells. 4. White blood cells. 5. Prednisolone.

2,5. 1. In 112, destruction of platelets is caused from what is believed to be an immune response, so giving additional platelets would only result in new platelets being destroyed. 2. Intravenous immunoglobulin is given because the cause of platelet destruction is believed to be an autoimmune response to disease-related antigens. Treatment is usually supportive. Activity is restricted at the onset because of the low platelet count and risk for injury that could cause bleeding. 3. Red blood cells are not an effective treatment for IP. 4. Because this is a platelet deficiency, white blood cells are not an effective treatment for ITP. White blood cell infusion is rarely done with any disease process.

1. How does the nurse interpret the laboratory analysis of a stool sample containing excessive amounts of azotorrhea and steatorrhea in a child with CF? The values indicate the child is: 1. Not compliant with taking her vitamins. 2. Not compliant with taking her enzymes. 3. Eating too many foods high in fat. 4. Eating too many foods high in fiber.

2. 1. 1. The child's compliance with vitamins would not be reflected in a laboratory result of azotorrhea and steatorrhea. If the client were not taking daily vitamin supple-ments, there might be a deficiency in those vitamins. 2. If the child were not taking enzymes, the result would be a large amount of undi-gested food, azotorrhea, and steatorrhea in the stool. Pancreatic ducts in CF clients become clogged with thick mucus that blocks the flow of digestive enzymes from the pancreas to the duodenum. Therefore, clients must take digestive enzymes with all meals and snacks to aid in absorption of nutrients. Often, teens are noncompliant with their medication regimen because they want to be like their peers. 3. Steatorrhea is an increased amount of fat in the stool. However, in CF patients, it is not a result of eating too many fatty foods. 4. Azotorrhea is an increased amount of protein in the stool. Steatorrhea is an increased amount of fat in the stool. Neither is a result of eating too many foods high in fiber.

2. Which would the nurse explain to parents about the inheritance of CF? 1. CF is an autosomal-dominant trait passed on from the child's mother. 2. CF is an autosomal-dominant trait passed on from the child's father. 3. The child of parents who are both carriers of the gene for CF has a 50% chance of acquiring CE. 4. The child of a mother who has CF and a father who is a carrier of the gene for CF has a 50% chance of acquiring CF

2. 4 1. CF is inherited as an autosomal-recessive trait. Both parents must be carriers of the gene in order for their child to inherit the disease. If both parents are carriers of the CF gene, each child they have has a 25% chance of acquiring the disease and a 50% chance of being a carrier of the dis-ease. If a child is born to a parent with CF and the other parent is a carrier, the child has a 50% chance of acquiring the disease and a 50% chance of being a carrier of the disease. 2. CF is inherited as an autosomal-recessive trait. Both parents must be carriers of the gene in order for their child to inheritthe gene. If a child is born to parents who are both carriers of the CF gene, the child has a 25% chance of acquiring the disease and a 50% chance of being a carrier of the disease. If the child is born to a parent with CF and the other parent is a carrier, the child has a 50% chance of acquiring the disease and a 50% chance of being a carrier of the disease. 3. If a child is born to parents who are both carriers of the CF gene, the child has a 25% chance of acquiring the disease and a 50% chance of being a carrier of the disease. 4. If the child is born to a parent with CF and the other parent is a carrier, the child has a 50% chance of acquiring the disease and a 50% chance of being a carrier of the disease.

21. Which foods would the nurse recommend to the mother of a 2-year-old with anemia? 1. 32 oz of whole cow's milk per day. 2. Meats, eggs, and green vegetables. 3. Fruits, whole grains, and rice. 4. 8 oz of juice, three times per day.

2. Meats, eggs, and green vegetables.

27. Which should the nurse instruct children to do to stop the spread of influenza in the classroom? Select all that apply. 1. Stay home if they have a runny nose and cough. 2. Wash their hands after using the restroom. 3. Wash their hands after sneezing. 4. Have a flu shot annuallv. 5. Drink lots of water during the day.

27. 2,3. 1. Children do not need to stay home unless they have a fever. However, the children should be taught to cough or sneeze into their elbow and to wash their hands after sneezing or coughing. 2. Children should always wash their hands after using the restroom. To decrease the spread of influenza, how-ever, it is more important for the children to wash their hands after sneezing or coughing. 3. It is essential that children wash their hands after any contact with nasopharyngeal secretions. 4. Children should have a flu shot annually, but that information is best included in an educational session for the parents. There is little that children can do directly to ensure they receive flu shots. Children of this age are often frightened of shots and would not likely pass that information on to their parents. 5. Drinking lots of water will keep children hydrated but not stop the spread of influenza.

4. The parent of an infant with CF asks the nurse how to meet the child's increased nutritional needs. Which is the nurse's best suggestion? 1. "You may need to increase the number of fresh fruits and vegetables you give your infant." 2. "You may need to advance your infant's diet to whole cow's mill because it is higher in fat than formula." "You may need to change your infant to a higher-calorie formula." 4. "You may need to increase your infant's carbohydrate intake."

3 1 Children with CF have difficulty absorbing nutrients because of the blockage of the pancreatic duct. Pancreatic enzymes cannot reach the duodenum to aid in digestion of food. These children often require up to 150% of the caloric intake of their peers. The nutritional recommendation for CF patients is a high-calorie and high-protein diet. 2. Whole cow's milk is a good source of fat but is not an increased source of protein that is recommended for CF patients. Another consideration is that whole cow's milk is not recommended until 12 months of age. 3. Often, infants with CF need to have a higher-calorie formula to meet their nutritional needs. Infants may also be placed on hydrolysate formulas that contain medium-chain triglycerides. 4. An increase in carbohydrate intake is not usually necessary. The nutritional recommendation for CF patients is a high-calorie and high-protein diet.

20. Which should the nurse administer to provide quick relief to a child with asthma who is coughing, wheezing, and having difficulty catching her breath? 1. Prednisone. 2. Montelukast (Singulair). 3. Albuterol. 4. Fluticasone (Flovent).

3 20. 1. Prednisone, a corticosteroid, is often given to children with asthma, but it is not a quick-relief medication. The prednisone will take time to relieve the child's symptoms. 2. Montelukast is an allergy medication that should be taken daily by people with asthma and significant allergies. Allergens are often triggers for asthma attacks, so treating the child for allergies can help avoid an attack. Montelukast, however, does not help a child immediately with the symptoms of a particular asthma attack. 3. Albuterol is the quick-relief bronchodilator of choice for treating an asthma attack. 4. Fluticasone is a long-term therapy medication for asthma and is used daily to help prevent asthma attacks.

31. Which statement by the parents of a toddler with repeated otitis media indicates they need additional teaching? 1. "If I quit smoking, my child may have a decreased chance of getting an ear infection." 2. "As my child gets older, he should have fewer ear infections, because his immune system will be more developed." 3. "My child will have fewer ear infections if he has his tonsils removed." 4. "My child may need a speech evaluation."

3 31. 1. Repeated exposure to smoke damages the cilia in the ear, making the child more prone to ear infections. 2. Children experience fewer ear infections as they age because their immune system is maturing. 3. Removing children's tonsils may not have any effect on their ear infections. Children who have repeated bouts of tonsillitis can have ear infections secondary to the tonsillitis, but there is no indication in this question that the child has a problem with tonsillitis. 4. Children who have repeated ear infections are at a higher risk of having decreased hearing during and between infections. Hearing loss directly affects a child's speech development.

33. Which should be included in instructions to the parent of a child prescribed amoxicillin to treat an ear infection? 1. "Continue the amoxicillin until the child's symptoms subside." 2. "Administer an over-the-counter antihistamine with the antibiotic.» 3. "Administer the amoxicillin until all the medication is gone." 4. "Allow your child to administer his own dose of amoxicillin."

3 33. 1. The parent should administer all of the medication. Stopping the medication when symptoms subside may not clear up the car infection and may actually cause more severe symptoms. 2. Antihistamines have not been shown to decrease the number of ear infections a child gets. 3. It is essential that all the medication be given. 4. The child is old enough to participate in the administration of medication but should do so only in the presence of the parents.

36. Which is the nurse's best response to the parent of an infant diagnosed with the first otitis media who wonders about long-term effects? 1. "The child could suffer hearing loss." 2. "The child could suffer some speech delays." 3. "The child could suffer recurrent ear infections." 4. "The child could require ear tubes

3 36. 1. Hearing loss is not an issue that would be discussed following one ear infection. Children with recurrent untreated ear infections are more likely to develop hearing loss. 2. Speech delays are not an issue that would be discussed following one ear infection. Children with recurrent untreated ear infections are more likely to develop some hearing loss, which often results in delayed language development. 3. When children acquire an ear infection at such a young age, there is an increased risk of recurrent infections. 4. Surgical intervention is not a first line of treatment. Surgery is usually reserved for children who have suffered from recurrent ear infections.

38. A parent asks the nurse how it will be determined whether their child has RSV. Which is the best response? 1. "We will do a simple blood test to determine whether your child has RSV." 2. "There is no specific test for RSV. The diagnosis is based on the child's symptoms." 3. "We will swab your child's nose and send that specimen for testing." 4. "We will have to send a viral culture to an outside laboratory for testing."

3 38. 1. RSV is not diagnosed by a blood test. 2. Nasal secretions are tested to determine whether a child has RSV. 3. The child is swabbed for nasal secre-tions. The secretions are tested to determine whether a child has RSV. 4. Viral cultures are not done very often because it takes several days to receive results. The culture does not have to be sent to an outside laboratory for evaluation.

A parent asks how to care for a child at home who has the diagnosis of viral tonsillitis. Which is the nurse's best response? 1. "You will need to give your child a prescribed antibiotic for 10 days." 2. "You will need to schedule a follow-up appointment in 2 weeks." 3. "You can give your child acetaminophen (Tylenol) every 4 to 6 hours as needed for pain." 4. "You can place warm towels around your child's neck for comfort."

3 24. 1. A viral illness does not require antibiotics. The patient would need to complete a course of antibiotics for bacterial tonsillitis. 2. Viral tonsillitis is usually a self-limiting disease and does not require a follow-up appointment unless the child's symptoms worsen. 3. Acetaminophen is recommended PRN for pain relief. 4. Warm compresses to the neck are not recommended, as they may increase the inflammation. Cold compresses or ice packs are recommended for comfort.

7. The parent of a child with CF asks the nurse what will be done to relieve the child's con-stipation. Which is the nurse's best response? 1. "Your child likely has an obstruction and will require surgery." 2. "Your child will likely be given IV fluids." 3. "Your child will likely be given MiraLAX." 4. "Your child will be placed on a clear liquid diet.

3 7. 1. CF clients who present with constipation usually do not require surgery. They commonly receive a stool softener or an osmotic solution orally to relieve their constipation. 2. IV fluids may be ordered if the client is NPO for any reason. However, IV fluids do not help relieve the client's constipation. CF clients with constipation commonly receive a stool softener or an osmotic solution orally to relieve their constipation. 3. CF clients with constipation commonly receive a stool softener or an osmotic solution such as polyethylene glycol 3350 (MiraLAX) orally to relieve their constipation. 4. CF clients are not placed on a clear liquid diet to relieve the constipation. CF clients with constipation commonly receive a stool softener or an osmotic solution orally to relieve their constipation. Once the constipation is relieved, the client will likely be placed on a low-fat diet and a stool softener.

3. The parent of a 4-month-old with CF asks the nurse what time to begin the child's first CPT each dav. Which is the nurse's best response? 1. "Thirty minutes before feeding the child breakfast." 2. "After deep-suctioning the child each morning." 3. "Thirty minutes after feeding the child breakfast." 4. "Only when the child has congestion or coughing."

3 1 3. 1. CPT should be done in the morning before feeding to avoid the risk of the child vomiting. 2. Infants with CF are not routinely deep-suctioned. Occasionally, if they have a weak cough reflex, infants may be suctioned following CPT to stimulate them to cough. 3. If CPT is done following feeding, it increases the likelihood that the child may vomit. 4. CPT should be done as a daily regimen with all CF patients. CPT helps to break up the secretions in the lungs and makes it easier for the patient to clear those secretions.

2. An 18-month-old male client is brought to the clinic by his mother. His height is in the50th percentile, and his weight is in the 80th percentile. The child is pale. The physical examination is normal, but his hematocrit level is 20%. Which of the following questions. Should assist the nurse in making a diagnosis? Select all that apply. 1. "How many bowel movements a dav does vour child have?" 2. "How much did your baby weigh at birth?" 3. "What does your child eat every day?" 4. "Has the child been given any new medications?" 5. "How much milk does your child drink per day?"

3, 5. 1. Because the child has a low hematocrit level, the child most likelv has anemia. Iron-deniciency anemia is the most common type of anemia. The number of bowel movements the child has 1s important information but not necessary to make the diagnosis of iron-deniciency anemia. Knowing birth weight can help determine whether the child is following his or her own curve on the growth chart. 3. A diet historv is necessary to determine the nutritional status of the child and whether the child is getting sufficient sources of iron. 4. Knowing if the child is taking any new med. ication is not necessary to make the diagno sis of iron-deficiency anemia 5. By asking how much milk the child con.sumes. une nurse can derermmne whetherthe child is riling up on milk and then not wanting to take food.

29 Which nursing action would help foster a hospitalized 3-year-old's sense of autonomy? 1. Let the child choose what time to take the oral antibiotics. 2. Allow the child to have a doll for medical play. 3. Allow the child to administer her own dose of Keflex (cephalexin) via oral syringe. 4. Let the child watch age-appropriate videos.

3. Allow the child to administer her own dose of Keflex (cephalexin) via oral syringe. Allowing preschoolers to participate in actions of which they are capable is an excellent way to enhance their autonomy.

35. The nurse realizes that a 3½-year-old's mother needs further education about the Denver Developmental Screening Test when she states: Select all that apply. 1. "It screens for gross motor skills." 2. "It screens for fine motor skills." 3. "It screens for intelligence level." 4. "It screens for language development." 5. "It screens for school readiness."

35. 3, 5 1. The Denver Developmental Screening Test evaluates children from 1 month to 6 years of age and is used to screen gross motor skills, fine motor skills, language development, and personal/social development. 2. The Denver Developmental Screening Test evaluates children from 1 month to 6 years and is used to screen gross motor skills, fine motor skills, language develop-ment, and personal/social development. 3. The Denver Developmental Screening Test does not test a child's level of intelligence. 4. The Denver Developmental Screening Test evaluates children from 1 month to 6 years and is used to screen gross motor skills, fine motor skills, language develop-ment, and personal/social development. 5. The Denver Developmental Screening Test evaluates children from 1 month to 6 years and is used to screen gross пото рини ане прото кіо (авивес. opment not school readiness.

37. An infant is not sleeping well, is crying frequently, has yellow drainage from the ear, and is diagnosed with an ear infection. Which nursing objective is the priority for the family? 1. Educating the parents about signs and symptoms of an ear infection. 2. Providing emotional support for the parents. 3. Providing pain relief for the child. 4. Promoting the flow of drainage from the ear.

37. 3 1. It is important to educate the family about the signs and symptoms of an ear infection, but that is not the priority at this time. The infant has already been diagnosed with the infection. 2. The parents may need emotional support because they are likely suffering from a lack of sleep because their infant is ill. However, this will not solve their current problems with their infant. 3. Providing pain relief for the infant is essential. With pain relief, the child will likely stop crying and rest better, as will the parents. 4. Promoting drainage flow from the ear is important, but providing pain relief is the highest priority.

39. A 3-year-old is attending her grandfather's funeral. Her parents told her that her grandfather is in heaven with God. Which statement describes a 3-year-old child's understanding of spirituality? 1. "The body is here with us on Earth, and the spirit is in heaven." 2. "He is in heaven. Is this heaven?" 3. "The spirit is no longer in his body." 4. "He won't need his body in heaven."

39. 1. Three-year-old children do not understand the difference between body and spirit. Their understanding of spirituality is literal in nature. ABSWER 2. Three-year-old children are literal thinkers. The child's parents told her that Grandpa was in heaven. She sees his body, so she thinks they are all in heaven. 3. Three-year-old children do not understand the difference between body and spirit. Their understanding of spirituality is literal in nature. 4. Three-year-old children think of spirituality in literal terms and do not understand the concept of heaven.

12. A child is complaining of throat pain. Which statement by the mother indicates that she needs more education regarding the care and treatment of her child's pharyngitis? 1. "I will have my child gargle with salt water three times a day." 2. "I will offer my child ice chips several times a day." 3. "I will give my child Tylenol every 4 to 6 hours as needed." 4. "I will ask the nurse practitioner for some amoxicillin.

4 12. 1. Gargling with warm salt water is a recommended treatment to relieve some of the discomfort associated with pharyngitis. 2. Encouraging ice chips is a recommended treatment to relieve some of the discomfort associated with pharyngitis. 3. Acetaminophen (Tylenol) is a suggested treatment for relief of discomfort related to pharyngitis. 4. Pharyngitis is a self-limiting viral illness that does not require antibiotic ther-apy. Pharyngitis should be treated with rest and comfort measures, including acetaminophen (Tylenol), throat sprays, cold liquids, and Popsicles.

15. The nurse is reviewing discharge instructions with the parents of a child who had a tonsillectomy a few hours ago. The parents tell the nurse that the child is a big eater, and they want to know what foods to give the child for the next 24 hours. What is the nurse's best response? 1. "The child's diet should not be restricted at all." 2. "The child's diet should be restricted to clear liquids." 3. "The child's diet should be restricted to ice cream and cold liquids." 4. "The child's diet should be restricted to soft foods."

4 15. 1. A child should be restricted to soft foods for the first couple of days postoperatively. Soft foods are recommended because the child will have a sore throat for several days following surgery. Soft foods will decrease the risk of bleeding. 2. Most children self-limit their food intake postoperatively. Children can have solids, but soft foods are recommended for the first several postoperative days. 3. Most children prefer to eat cold foods, but they are not restricted to them. 4. Soft foods are recommended to limit the child's pain and to decrease the risk for bleeding.

23. The parents of a 6-year-old who has a new diagnosis of asthma ask the nurse what to do to make their home a more allergy-free environment. Which is the nurse's best response? 1. "Use a humidifier in your child's room." 2. "Have your carpet cleaned chemically once a month." 3. "Wash household pets weekly." 4. "Avoid purchasing upholstered furniture."

4 23. 1. It is best to maintain 30% to 50% humidity in homes of children with asthma. However, humidifiers are not recommended because they can harbor mold if they are not cleaned properly. 2. Chemical cleaning is not recommended because the chemicals used can be a trigger and actually cause the child to have an asthma attack. The best recommendation is to remove all carpet from the house, if possible. 3. Household pets are not recommended for children with asthma. 4. Leather furniture is recommended rather than upholstered furniture. Upholstered furniture can harbor large amounts of dust, whereas leather furniture may be wiped off regularly with a damp cloth.

24. Which of the tollowing measures should the nurse teach the parent of a child with hemophilia to do first if the child sustains an iniury to a joint causing bleeding: 1. Give the child a dose of acetaminophen (Tylenol). 2. Immobilize the joint and elevate the extremity. 3. Apply neat to the area. 4 Administer tactor per the home-care protocol.

4 24. 1. Acetaminophen (Tylenol) helps with the 4. Inbe pain but does not stop the bleeding. dest 2. Elevating and immobilizing the extremity anen are good interventions as they decrease and with blood flow. Factor should be administered Can first, however. mal 3. Cold, not heat, should be applied to pro- TEST-TAKIN mote vasoconstriction. ogy of the 4. Administration of factor should be the tion, the e first intervention if home-care transfu- be identif sions have been initiated.

25. Which comment should the parent of a 2-year-old expect from the toddler about a new baby brother? 1. "When the baby takes a nap, will you play with me?" 2. "Can I play with the baby?" 3. "The baby is so cute. I love him." 4. "It is time to put him away so we can play."

4. "It is time to put him away so we can play." This is a typical statement that would be made by a toddler. Toddlers are very egocentric and do not consider the needs of the other child.

32. Which approach should the nurse use to gather information from a child brought to the ED for suspected child abuse? 1. Promise the child that her parents will not know what she tells the nurse. 2. Promise the child that she will not have to see the suspected abuser again. 3. Use correct anatomical terms to discuss body parts. 4. Tell the child that the abuse is not her fault and that she is a good person.

4. Tell the child that the abuse is not her fault and that she is a good person.

24. According to developmental theories, which important event is essential to the development of the toddler? 1. The child learns to feed self. 2. The child develops friendships. 3. The child learns to walk. 4. The child participates in being potty-trained.

4. The child participates in being potty-trained. Developmental theorists like Erickson and Freud believe that toilet training is the essential event that must be mastered by the toddler.

41. Which nursing action is most appropriate to gain information about how a child is feeling? 1. Actively attempt to make friends with the child before asking about her feelings. 2. Ask the child's parents what feelings she has expressed in regard to her diagnosis. 3. Provide the child with some paper to draw a picture of how she is feeling. 4. Ask the child direct questions about how she is feeling.

41. 1. The nurse should not attempt to make friends with the child too quickly. The child should be given the opportunity to observe the nurse working in order to increase her comfort level with the nurse. 2The child's parents are a good source of information, but the child may not have expressed all of her feelings to her parents. ANSWER 3. Children often include much more detail about their feelings in drawings than in words, expressing things in pictures that they are unable to verbalize. 4. School-age children do not often share all of their feelings verbally, especially to people with whom they are not familiar.

43. Which statement accurately describes how the nurse should approach an 11-year-old to do a physical assessment? 1. Ask the child's parents to remain in the room during the physical exam. 2. Auscultate the heart, lungs, and abdomen first. 3. Explain that the physical exam will not hurt. 4. Explain what the nurse will be doing in basic understandable terms.

43. 1. Privacy is very important to school-age children. The child should be given the choice of having his parents present for the exam. 2. School-age children can be assessed in a head-to-toe sequence. 3. The nurse should not promise that the exam will not hurt. ANSWER 4. School-age children are capable of understanding basic functions of the body and can understand what the nurse will be doing if explained in basic terms.

44. Which is the best method of distraction for an 8-vear-old who is having surgery later today to insert a central line and is NPO? 1. Use the telephone to call friends. 2. Watch television. 3. Play a board game. 4. Read the central-line pamphlet he was given.

44. 1. Talking to friends may distract the child for some time. However, the conversation could revert to a discussion about the upcoming surgery. 2. Watching television may distract the child for some time, but he may still be thinking about his surgery. ANS 3 A board game is the optimal choice because school-age children enjoy being engaged with others in an activity that requires some skill and challenge. 4. Reading material about the surgery will only increase his thoughts about the surgery.

45. Which activity can the nurse provide for a 9-year-old to encourage a sense of industry? 1. Allow the child to choose what time to take his medication. 2. Provide the child with the homework his teacher has sent. 3. Allow the child to assist with his bath. 4. Allow the child to help with his dressing change.

45. 1. Giving the child choices while in the hospital is important. However, medications should be kept on schedule. It is essential to give them at the prescribed time. ANS 2 The school-age child is focused on academic performance; therefore, the child can achieve a sense of industry by completing his homework and staying on track with his classmates. 3. The child should have already mastered bathing. It is not likely to give him a sense of accomplishment. 4. The child may enjoy assisting with his dressing change, but it is not the best example of industry.

6. A 2-vear-old has just been diagnosed with CF. The parents ask the nurse what early respiratory symptoms they should expect to see in their child. Which is the nurse's best response? 1. "You can expect your child to develop a barrel-shaped chest." 2. "You can expect your child to develop a chronic productive cough." 3. "You can expect your child to develop bronchiectasis." 4. "You can expect your child to develop wheezing respirations."

6. 4 1. A barrel-shaped chest is a long-term respiratory problem that occurs as a result of recurrent hyperinflation of alveoli. 2. A chronic productive cough is common as pulmonary damage increases. 3. Bronchiectasis develops in advanced stages of CF. 4. Wheezing respirations and a dry, nonproductive cough are common early symptoms in CF.

63 Treatment for a 4-year-old with HSP includes which of the following? Select all that apply. 1. Administer NSAIDS for pain. 2. Monitor blood pressure. 3. Monitor for renal involvement. 4. Position of comfort for joint pain. 5. Apply a cream for the maculopapular rash.

63. 1, 2, 3. 1. NSAIDs such as ibuprofen or naproxen are recommended for discomfort and pain. 2. Monitor blood pressure because many have gastrointestinal involvement with inflammation and hemorrhage. 3. Half of all children with HSP have renal involvement, which is the most serious long-term complication. 4. The child's limbs should be in anatomically correct position not position of comfort. 5. The rash does not require any type of cream.

65 Which are the common causes of acquired aplastic anemia? Select all chat apply. 1. Certain anticonvulsants medications 2. Certain chemotherapeutic agents. 3. Idiopathic causes. 4 Prednisolone 5 Indoor wall paints.

65. 1, 2, 3. 1. Common causes of acquired aplastic anemia include certain anticonvulsant medications. 2. Common causes of acquired aplastic anemia include certain chemotherapeutic agents. 3. The most common cause of acquired aplastic anemia is idiopathic, in other words, the cause is not known. 4. Prednisolone is not known to cause acquired aplastic anemia. 5. Indoor wall paints are not known to cause acquired aplastic anemia, but paint removers and lacquers can cause it.

7. Which of the following activities should a nurse suggest for a client diagnosed with hemophilia? Select all that apply 1. Swimming 2. Golf. 3. Hiking 4. Fishing. 5. Soccer

7. 1, 2, 3, 4. 1. Children with hemophilia should be encouraged to take part in noncontact activities that allow for social, psycho-logical, and physical growth, such as swimming. 2. Children with hemophilia should be encouraged to take part in noncontact activities that allow for social, psycholog-ical, and physical growth, such as golf. 3. Children with hemophilia should be encouraged to take part in noncontact activities that allow for social, psycholog-ical, and physical growth, such as hiking. 4. Children with hemophilia should be encouraged to take part in noncontact activities that allow for social, psycholog-ical, and physical growth, such as fishing. 5. Contact sports such as soccer should be discouraged.

8. Which of the following describes IT? Select all that apply. 1. ITP is a congenital hematological disorder. 2. ITP causes excessive destruction of platelets 3. Children with ITP have normal bone marrow. 4. Platelets are small in ITP. S. Petechiae are common in ITP

8. 2,3, 5. 1. IT is an acquired hematological condition that is characterized by excessive destruction of platelets, purpura, and normal bone marrow along with an increase in large, yellow platelets. 2. IT is characterized by excessive destruction of platelets. 3. The bone marrow is normal in children with ITP. 4. ICP would fricansidered if the child had symptoms of headache, nausea, forcefiat fomiting, blurred or double vision, drows. iness, or seizure; an IP might then be performed. 5. Leukemia is not staged.

8. The parents of a 5-week-old have just been told that their child has CF. The mother had a sister who died of CF when she was 19 years of age. The parents are sad and ask the nurse about their child's projected life expectancy. What is the nurse's best response? 1. "The life expectancy for CF patients has improved significantly in recent years." 2. "Your child might not follow the same course that the mother's sister did." 3. "The health-care provider will come to speak to vou about treatment options." 4. The nurse answers their questions briefly, listens to their concerns, and is available later after they've processed the information

8. 4 1. The parents are devastated by the new diagnosis and are likely not ready to hear about the current life expectancy for CF patients. They are in shock and are trying to deal with the new diagnosis, so any additional information will increase their stress level. The mother had a negative experience with CF in her own family and will likely continue to focus on her past experience with the disease. It may be more effective to give her time to consider her child's diagnosis and present her with information about current treatments and life span in a few days. 3. Having the health-care provider return with additional information will not help these parents at this time. They need time before they are given additional information. 4. The nurse's best intervention is to listen as the parents express their concerns and fears. The nurse's answers should be brief, knowing that the parents are not able to process information right now. The nurse should be available later if the parents have any other concerns or questions or if they just need someone with whom to talk.

9. The nurse is caring for a child who is receiving a transfusion of PRBCs. The nurse is aware that it the child has a hemolytic reaction to the blood, the signs and symptoms would include which of the following? Select all that apply. 1. Fever. 2. Rash. 3. Oliguria. 4. Hypotension 5. Chills.

9. 1, 3, 4. 1. Hemolytic reactions include fever, pain at insertion site, hypotension, renal fail. ure, tachycardia, oliguria, and shock. 2. Febrile reactions are fever, rash, and chills. Allergic reactions include hives, itching, and respiratory distress. 3. Hemolytic reactions include fever, pain at insertion site, hypotension, renal fail. ure, tachycardia, oliguria, and shock. 4. Hemolytic reactions include fever, pain at insertion site, hypotension, renal fail. ure, tachycardia, oliguria, and shock. 5. Febrile reactions are fever, chills, and rash Allergic reactions include rash, hives, and respiratory distress.

9. A 7-month-old has a low-grade fever, nasal congestion, and a mild cough. What should the nursing care management of this child include? Select all that apply. 1. Maintaining strict bedrest. 2. Avoiding contact with family members. 3. Instilling saline nose drops and bulb suctioning. 4. Keeping the head of the bed flat. 5. Providing humidity and propping the head of the bed up.

9. 3, 5. 1. Strict bedrest is not necessary. Children with respiratory illnesses usually self-limit their activity. Parents just need to ensure that their children are getting adequate rest. 2. It is not necessary to avoid contact with family members. Nasopharyngitis is spread by contact with the secretions, so hand washing is key to limiting the spread of the virus. 3. Infants are nose breathers and often have problems breathing when congested. Nasal saline drops and gentle suctioning with a bulb syringe are often recommended. 4. The head of the bed should be elevated to help with the drainage of secretions. 5. Other helpful suggestions for infants with colds are to provide humidity with a cool mist humidifier or take them into a steamy bathroom; in addition, raising the head of the bed helps with drainage of secretions.

52. Which of the following is the expected urine output for a school-age child? 1. 1 mL/kg/hr. 2. 3 mL/kg/hr. 3. 4 mL/kg/hr. 4. 5 mL/kg/hr.

ANS 1. A school-age child can expect to have a urine output of 1 ml./g/hr. 2. A school-age child can expect to have a urine output of 1 mL./kg/hr. . A school-age child can expect to have a urine output of 1 mL/kg/hr. 4. A school-age child can expect to have a urine output of 1 ml./kg/hr.


संबंधित स्टडी सेट्स

Agency 3 - Real Estate Brokerage Compensation

View Set

The Effects of Drugs on the Brain

View Set

Multiple Samples Assignment 100%

View Set

American Government Straighterline Test 1

View Set

Information Systems- Ch. 9-10, 13-15 Quiz

View Set

Nursing Care of Children with Special Health Care Needs

View Set

Understanding Ultrasound Physics FINAL w/ Formulas

View Set